Topic: Uterine Supports
Subtopic: Ligaments of the Uterus
Keyword Definitions:
• Uterine ligaments: Fibromuscular structures supporting uterus in pelvic cavity.
• Transverse cervical ligament (Cardinal ligament): Extends from cervix and lateral vagina to lateral pelvic wall.
• Pubocervical ligament: Connects cervix to pubic bone anteriorly.
• Round ligament: Maintains anteversion of uterus, runs from uterus to labia majora.
Lead Question - 2014
Ligament extending from cervix and vagina to lateral pelvic wall?
a) Broad ligament
b) Pubocervical ligament
c) Round ligament
d) Transverse cervical ligament
Explanation:
The correct answer is d) Transverse cervical ligament. Also known as the cardinal ligament, it provides major support to the uterus and cervix, extending laterally from the cervix and vagina to the pelvic wall. It carries uterine vessels and resists uterine descent. Weakening causes uterine prolapse or cervical descent clinically.
1. Which ligament maintains the anteverted position of the uterus?
a) Broad ligament
b) Round ligament
c) Uterosacral ligament
d) Pubocervical ligament
Explanation:
The correct answer is b) Round ligament. The round ligament extends from the uterine fundus to the labia majora through the inguinal canal. It maintains the anteverted and anteflexed position of the uterus, preventing its backward tilt. During pregnancy, it stretches considerably and may cause “round ligament pain.”
2. The broad ligament of the uterus is a fold of:
a) Peritoneum
b) Endopelvic fascia
c) Connective tissue
d) Muscular tissue
Explanation:
The correct answer is a) Peritoneum. The broad ligament is a double layer of peritoneum extending from the sides of the uterus to the lateral pelvic walls. It acts as a mesentery for the uterus and contains the uterine tube, round ligament, ovarian ligament, and uterine vessels within its layers.
3. Clinical case: A 45-year-old woman with uterine prolapse likely has weakening of which ligament?
a) Broad ligament
b) Cardinal ligament
c) Round ligament
d) Pubococcygeus muscle
Explanation:
The correct answer is b) Cardinal ligament. The cardinal or transverse cervical ligament provides strong lateral support to the cervix and uterus. Its weakening leads to descent of the cervix and uterus through the vaginal canal, resulting in uterine prolapse. Pelvic floor muscle laxity may worsen this condition.
4. Uterosacral ligament connects:
a) Cervix to sacrum
b) Fundus to sacrum
c) Cervix to pubic bone
d) Vagina to ischial spine
Explanation:
The correct answer is a) Cervix to sacrum. The uterosacral ligaments are paired fibromuscular structures extending from the posterior cervix to the sacrum. They maintain the uterus in an anteverted position and prevent its posterior displacement. They are palpable during pelvic examination and important in surgical repairs of prolapse.
5. Clinical case: During a hysterectomy, which ligament must be carefully ligated to avoid ureteric injury?
a) Round ligament
b) Cardinal ligament
c) Uterosacral ligament
d) Broad ligament
Explanation:
The correct answer is b) Cardinal ligament. The uterine artery runs through the cardinal ligament, crossing the ureter about 2 cm lateral to the cervix. During ligation of the uterine artery in hysterectomy, surgeons must carefully identify and preserve the ureter to prevent accidental injury or ureteric obstruction.
6. Pubocervical ligaments extend between:
a) Cervix and pubic bone
b) Cervix and sacrum
c) Cervix and lateral pelvic wall
d) Cervix and ischial spine
Explanation:
The correct answer is a) Cervix and pubic bone. The pubocervical ligaments support the bladder neck and anterior vaginal wall by connecting the cervix and upper vagina to the posterior surface of the pubic bone. Their weakening leads to anterior vaginal wall prolapse (cystocele) and urinary incontinence in women.
7. Clinical case: A woman with pelvic pain and uterine retroversion likely has laxity of which structure?
a) Round ligament
b) Broad ligament
c) Uterosacral ligament
d) Pubocervical ligament
Explanation:
The correct answer is c) Uterosacral ligament. The uterosacral ligament maintains the anteverted position of the uterus. Its laxity allows the uterus to tilt posteriorly (retroversion), resulting in pelvic discomfort, backache, and dyspareunia. Strengthening these ligaments surgically may restore the normal position of the uterus.
8. The ligament containing the uterine artery is:
a) Round ligament
b) Broad ligament
c) Cardinal ligament
d) Uterosacral ligament
Explanation:
The correct answer is c) Cardinal ligament. The uterine artery runs within the cardinal ligament, supplying blood to the uterus. This ligament extends from the cervix and upper vagina to the lateral pelvic wall. It provides major support and stability to the uterus in the pelvic cavity.
9. Clinical case: Following childbirth, a patient develops cystocele. Which ligament is likely damaged?
a) Pubocervical ligament
b) Uterosacral ligament
c) Round ligament
d) Cardinal ligament
Explanation:
The correct answer is a) Pubocervical ligament. The pubocervical ligament supports the bladder neck and anterior vaginal wall. During childbirth, excessive stretching or tearing weakens this ligament, leading to herniation of the bladder into the vagina (cystocele). Surgical repair aims to restore pelvic support and bladder function.
10. Which structure forms the main mechanical support for the uterus?
a) Broad ligament
b) Pelvic diaphragm
c) Uterosacral and cardinal ligaments
d) Round ligament
Explanation:
The correct answer is c) Uterosacral and cardinal ligaments. Together, these ligaments form the main mechanical support for the uterus, maintaining its central position and preventing descent. They attach the cervix to the sacrum and pelvic walls, respectively, forming a supportive sling around the cervix and upper vagina.
Subtopic: Vaginal Anatomy and Relations
Keyword Definitions:
Vagina: A fibromuscular canal extending from the cervix to the vulva, forming part of the female genital tract.
Posterior vaginal wall: The back wall of the vagina, longer than the anterior wall, related to the rectouterine pouch.
Hymen: A mucous membrane fold that partially covers the vaginal opening in virgins.
Vaginal fornices: Recesses around the cervix formed by the vaginal wall.
Lead Question (2014): Length of posterior vaginal wall is
a) Variable
b) Same as anterior vaginal wall
c) Less than anterior vaginal wall
d) More than anterior vaginal wall
Explanation: The posterior vaginal wall is about 9 cm long, which is longer than the anterior vaginal wall (approximately 7.5 cm). This difference exists because the vagina slopes upward and backward to the cervix. Answer: d) More than anterior vaginal wall
1. The average length of the vagina in adult females is?
a) 5–6 cm
b) 7–8 cm
c) 8–10 cm
d) 10–12 cm
Explanation: The average vaginal length in adult females is about 8–10 cm, with the posterior wall being slightly longer than the anterior. Answer: c) 8–10 cm
2. The posterior vaginal wall is related to which structure superiorly?
a) Rectouterine pouch
b) Urinary bladder
c) Urethra
d) Ureter
Explanation: Superiorly, the posterior vaginal wall is related to the rectouterine pouch (of Douglas), an important peritoneal space clinically significant for fluid collection. Answer: a) Rectouterine pouch
3. The vaginal blood supply mainly comes from?
a) Ovarian artery
b) Vaginal artery
c) Inferior epigastric artery
d) Uterine artery only
Explanation: The vagina receives its main blood supply from the vaginal artery, a branch of the internal iliac artery, with contributions from the uterine and internal pudendal arteries. Answer: b) Vaginal artery
4. A 32-year-old woman presents with fluid in the rectouterine pouch. This can be accessed through?
a) Posterior fornix of vagina
b) Anterior fornix of vagina
c) Cervical canal
d) Urethra
Explanation: The rectouterine pouch can be accessed surgically or diagnostically through the posterior fornix of the vagina for drainage or sampling. Answer: a) Posterior fornix of vagina
5. The epithelium lining the vagina is?
a) Stratified squamous non-keratinized
b) Simple cuboidal
c) Transitional
d) Ciliated columnar
Explanation: The vaginal epithelium is stratified squamous non-keratinized, providing resistance to friction during intercourse and childbirth. Answer: a) Stratified squamous non-keratinized
6. The nerve supply to the lower one-third of the vagina is?
a) Pudendal nerve
b) Pelvic splanchnic nerve
c) Hypogastric plexus
d) Sacral splanchnic nerve
Explanation: The lower one-third of the vagina is supplied by the pudendal nerve, which carries somatic sensory fibers, making it sensitive to pain. Answer: a) Pudendal nerve
7. During childbirth, the posterior vaginal wall may tear due to?
a) Prolonged second stage
b) Posterior position of head
c) Large fetal head
d) All of the above
Explanation: Posterior vaginal wall tears occur due to overdistension or trauma from a large fetal head or prolonged second stage of labor. Answer: d) All of the above
8. The anterior vaginal wall is related to which organ?
a) Rectum
b) Urinary bladder
c) Sigmoid colon
d) Uterus
Explanation: The anterior vaginal wall is related to the urinary bladder and urethra, which are closely apposed to it. Answer: b) Urinary bladder
9. A posterior colpotomy is performed through which part of vagina?
a) Posterior fornix
b) Anterior fornix
c) Vaginal vault
d) Cervix
Explanation: A posterior colpotomy is done through the posterior fornix of the vagina to access the peritoneal cavity for drainage or sterilization procedures. Answer: a) Posterior fornix
10. The vaginal wall lacks which of the following?
a) Serosa
b) Mucosa
c) Muscular layer
d) Adventitia
Explanation: The vaginal wall has mucosa, muscular, and adventitial layers but lacks a serosa, as it is not covered by peritoneum except at the posterior fornix. Answer: a) Serosa
Topic: Nipple and Areola
Subtopic: Lactiferous Ducts
Keyword Definitions:
Lactiferous ducts: Channels that carry milk from mammary glands to the nipple.
Nipple: The conical projection in the center of the areola where ducts open externally.
Areola: Pigmented circular skin around the nipple containing sebaceous glands.
Mammary gland: Modified sweat gland specialized in milk secretion.
Lobules: Functional units of breast, drained by lactiferous ducts.
Lead Question - 2014
How many lactiferous ducts open in nipple ?
a) 0 -10
b) 15 -20
c) 25 -50
d) 50 -75
Explanation: Each breast usually has 15 to 20 lactiferous ducts, each draining a separate lobe. These ducts converge and open independently on the nipple surface to deliver milk during lactation. Thus, the correct answer is 15–20.
Guessed Questions:
1) Which structure surrounds the openings of lactiferous ducts?
a) Areola
b) Sebaceous glands
c) Montgomery’s tubercles
d) Nipple
Explanation: The nipple contains the terminal openings of lactiferous ducts. It is surrounded by the areola, which houses sebaceous glands (Montgomery’s glands) that lubricate and protect during breastfeeding. Correct answer is Nipple.
2) A lactating mother presents with blockage of a single duct causing localized swelling. Which structure is obstructed?
a) Alveolus
b) Lactiferous sinus
c) Lactiferous duct
d) Areolar gland
Explanation: Obstruction of a lactiferous duct prevents drainage of milk from the corresponding lobe, producing localized swelling and pain. This condition can lead to mastitis if untreated. Correct answer is Lactiferous duct.
3) Which hormone primarily stimulates milk secretion from breast lobules?
a) Estrogen
b) Progesterone
c) Prolactin
d) Oxytocin
Explanation: Prolactin secreted from the anterior pituitary promotes milk synthesis in alveolar cells of breast lobules, while oxytocin causes milk ejection through ducts. Correct answer is Prolactin.
4) Which hormone is responsible for milk ejection reflex during suckling?
a) Estrogen
b) Prolactin
c) Oxytocin
d) Progesterone
Explanation: Oxytocin from the posterior pituitary causes contraction of myoepithelial cells around alveoli and ducts, resulting in milk letdown reflex during suckling. Correct answer is Oxytocin.
5) A 25-year-old lactating woman presents with cracked nipple and mastitis. Infection spreads most likely through?
a) Areolar glands
b) Lactiferous ducts
c) Lymphatic vessels
d) Intercostal nerves
Explanation: Infection of the breast often enters through cracks in the nipple and spreads via lactiferous ducts into glandular tissue, leading to mastitis. Correct answer is Lactiferous ducts.
6) Which type of gland is the mammary gland?
a) Apocrine gland
b) Holocrine gland
c) Merocrine gland
d) Modified sweat gland
Explanation: Mammary gland is a modified sweat gland of apocrine type, functioning in milk production under hormonal regulation. Correct answer is Modified sweat gland.
7) A newborn fails to suckle effectively. Which structure in the mother provides direct milk entry to the infant?
a) Areola
b) Lactiferous ducts
c) Nipple
d) Lobules
Explanation: The nipple is the direct structure delivering milk from lactiferous ducts to the infant’s mouth. Ineffective suckling prevents proper milk ejection. Correct answer is Nipple.
8) Which artery mainly supplies blood to the lactating breast?
a) Internal thoracic artery
b) Radial artery
c) Brachial artery
d) Axillary artery
Explanation: The breast receives blood supply mainly from perforating branches of internal thoracic artery, lateral thoracic artery, and intercostal arteries. Correct answer is Internal thoracic artery.
9) A 40-year-old woman presents with retracted nipple. This is commonly due to?
a) Obstruction of ducts
b) Fibrosis of lactiferous ducts
c) Blockage of lymphatics
d) Muscular spasm
Explanation: Carcinoma of the breast causes fibrosis and retraction of lactiferous ducts, leading to nipple retraction. Correct answer is Fibrosis of lactiferous ducts.
10) Which lymph nodes primarily drain the nipple and areola?
a) Parasternal nodes
b) Supraclavicular nodes
c) Axillary nodes
d) Infraclavicular nodes
Explanation: The axillary lymph nodes, especially anterior (pectoral) group, are the primary drainage site for nipple and areola, making them crucial in breast cancer staging. Correct answer is Axillary nodes.
Chapter: Reproductive Physiology
Topic: Male Reproduction
Subtopic: Spermatogenesis
Keyword Definitions:
• Spermatogenesis: Process by which spermatozoa develop from spermatogonia.
• Spermiogenesis: Transformation of spermatids into spermatozoa.
• Spermatogonia: Stem cells of seminiferous tubules.
• Sertoli cells: Supporting cells aiding spermatogenesis.
• Leydig cells: Testosterone-secreting cells of testes.
• Epididymis: Site for sperm storage and maturation.
• Seminiferous tubules: Functional units where spermatogenesis occurs.
• Ductus deferens: Sperm transport tube.
• Prostate: Produces seminal fluid component.
• Acrosome: Cap of sperm head containing enzymes.
Lead Question - 2013
Spermatogenesis takes place in ?
a) Epididymis
b) Seminiferous tubule
c) Ductus deferens
d) Prostate
Explanation: Spermatogenesis, the process of sperm production, occurs inside the seminiferous tubules of testes. Sertoli cells support and nourish developing sperm, while Leydig cells produce testosterone. Epididymis stores sperm, but formation happens only in seminiferous tubules. Correct answer: b) Seminiferous tubule.
1) Which hormone is essential for initiating spermatogenesis?
a) LH
b) FSH
c) Prolactin
d) ACTH
Explanation: FSH acts on Sertoli cells to initiate spermatogenesis. LH primarily stimulates Leydig cells to produce testosterone. Together, both regulate sperm formation. Correct answer: b) FSH.
2) Clinical case: A young male with hypogonadotropic hypogonadism presents with azoospermia. Which hormone therapy will help restore spermatogenesis?
a) Prolactin
b) GH
c) FSH and LH
d) Cortisol
Explanation: Both FSH and LH are required. FSH stimulates Sertoli cells, while LH stimulates Leydig cells for testosterone. Their combination restores spermatogenesis. Correct answer: c) FSH and LH.
3) Duration of complete spermatogenesis in humans is approximately:
a) 24 hours
b) 24 days
c) 64 days
d) 120 days
Explanation: Spermatogenesis from spermatogonia to mature spermatozoa takes about 64 days. This includes mitosis, meiosis, and spermiogenesis. Correct answer: c) 64 days.
4) Clinical case: A patient with Klinefelter syndrome is infertile. Which abnormality in spermatogenesis is expected?
a) Normal spermatid formation
b) Absence of spermatogenesis
c) Increased sperm count
d) Normal motility
Explanation: Klinefelter syndrome (47,XXY) leads to seminiferous tubule fibrosis, small testes, and absent spermatogenesis, causing infertility. Correct answer: b) Absence of spermatogenesis.
5) The blood-testis barrier is formed by:
a) Leydig cells
b) Sertoli cells
c) Spermatogonia
d) Myoid cells
Explanation: Sertoli cells form tight junctions creating the blood-testis barrier, protecting developing spermatocytes from immune recognition. Correct answer: b) Sertoli cells.
6) Clinical case: A man receiving testosterone injections for bodybuilding develops infertility. The reason is:
a) Stimulation of FSH
b) Inhibition of GnRH, FSH, and LH
c) Increased Sertoli cell activity
d) Increased spermiogenesis
Explanation: Exogenous testosterone suppresses hypothalamic GnRH and pituitary FSH/LH secretion, reducing intratesticular testosterone and halting spermatogenesis. Correct answer: b) Inhibition of GnRH, FSH, and LH.
7) Which stage of spermatogenesis is haploid?
a) Spermatogonia
b) Primary spermatocyte
c) Secondary spermatocyte
d) Spermatogonia type B
Explanation: Secondary spermatocytes and spermatids are haploid, formed after meiosis I. Correct answer: c) Secondary spermatocyte.
8) Clinical case: A chemotherapy patient presents with permanent azoospermia. Which testicular cells are most sensitive to chemotherapy?
a) Spermatogonia
b) Sertoli cells
c) Leydig cells
d) Spermatozoa
Explanation: Spermatogonia are highly mitotically active and most vulnerable to chemotherapy and radiation, leading to infertility. Correct answer: a) Spermatogonia.
9) Capacitation of sperm occurs in:
a) Testis
b) Epididymis
c) Female genital tract
d) Prostate
Explanation: Capacitation, essential for fertilization, occurs in the female reproductive tract, making sperm capable of acrosome reaction. Correct answer: c) Female genital tract.
10) Clinical case: A male with immotile sperm but normal morphology and count likely has defect in:
a) Mitochondrial sheath
b) Dynein arms of flagella
c) Acrosome
d) Sertoli cell function
Explanation: Dynein arms are required for flagellar movement. Their absence causes immotile sperm syndrome (Kartagener syndrome), leading to infertility. Correct answer: b) Dynein arms of flagella.
Topic: Spermatogenesis
Subtopic: Spermiogenesis
Keyword Definitions:
• Spermatogenesis: Process of sperm production from spermatogonia.
• Spermiogenesis: Final stage where spermatids transform into spermatozoa.
• Spermatogonia: Diploid stem cells in seminiferous tubules.
• Primary spermatocyte: Cell undergoing meiosis I.
• Secondary spermatocyte: Product of meiosis I, haploid cells.
• Spermatid: Haploid, immature sperm precursor cell.
• Spermatozoa: Mature motile male gametes.
• Sertoli cells: Support spermatogenesis and secrete inhibin.
• Leydig cells: Secrete testosterone under LH stimulation.
• Acrosome: Cap-like vesicle in sperm, helps penetration of ovum.
Lead Question - 2013
Spermiogenesis refers to ?
a) Formation of spermatazoa from spermatogonia
b) Formation of spermatazoa from spermatids
c) Formation of spermatids from spermatocytes
d) Formation of secondary spermatocytes from primary spermatocytes
Explanation: Spermiogenesis is the final phase of spermatogenesis, in which haploid spermatids undergo morphological changes to form spermatozoa. Changes include acrosome formation, condensation of nucleus, development of flagellum, and shedding of cytoplasm. This transformation does not involve cell division. Correct answer: b) Formation of spermatozoa from spermatids.
1) Which hormone stimulates Leydig cells to produce testosterone?
a) FSH
b) LH
c) Prolactin
d) Inhibin
Explanation: LH (Luteinizing Hormone) stimulates Leydig cells in the testes to secrete testosterone, which is essential for spermatogenesis and development of male secondary sexual characters. FSH stimulates Sertoli cells. Correct answer: b) LH.
2) Clinical case: A man with infertility has decreased sperm count but normal testosterone. Likely defect is in:
a) Leydig cells
b) Sertoli cells
c) Hypothalamus
d) Adrenal cortex
Explanation: Sertoli cells nourish spermatids, regulate spermiogenesis, and secrete inhibin. Infertility with normal testosterone suggests Sertoli cell dysfunction. Correct answer: b) Sertoli cells.
3) Acrosome of sperm is derived from:
a) Nucleus
b) Mitochondria
c) Golgi apparatus
d) Rough ER
Explanation: The acrosome is a cap-like structure covering the anterior part of the sperm head. It originates from the Golgi apparatus and contains enzymes like hyaluronidase to help penetration of ovum. Correct answer: c) Golgi apparatus.
4) Clinical case: A patient with pituitary adenoma shows low FSH but normal LH. Which function will be most affected?
a) Testosterone secretion
b) Spermatogenesis
c) Secondary sex character development
d) Libido
Explanation: FSH acts on Sertoli cells to support spermatogenesis. Low FSH impairs spermatogenesis despite normal testosterone. Correct answer: b) Spermatogenesis.
5) The mitochondrial sheath of sperm is located in:
a) Head
b) Middle piece
c) Tail
d) Acrosome
Explanation: Mitochondria form a spiral sheath around the middle piece of sperm, providing ATP for motility. Correct answer: b) Middle piece.
6) Clinical case: A man with mutation in dynein arms of sperm shows infertility due to:
a) Acrosomal defect
b) Lack of motility
c) Defective binding to ovum
d) Spermatid arrest
Explanation: Dynein arms are required for flagellar movement. Their absence causes immotile sperm, leading to infertility. This condition is seen in Kartagener’s syndrome. Correct answer: b) Lack of motility.
7) Which cells form the blood-testis barrier?
a) Leydig cells
b) Sertoli cells
c) Spermatogonia
d) Myoid cells
Explanation: Sertoli cells are connected by tight junctions that form the blood-testis barrier, protecting developing spermatogenic cells from autoimmune attack. Correct answer: b) Sertoli cells.
8) Clinical case: A male undergoing chemotherapy develops azoospermia. Which cell type is most susceptible?
a) Spermatogonia
b) Spermatids
c) Sertoli cells
d) Leydig cells
Explanation: Spermatogonia are highly mitotically active, making them most sensitive to chemotherapy and radiation. Their destruction leads to azoospermia. Correct answer: a) Spermatogonia.
9) During spermiogenesis, excess cytoplasm is shed as:
a) Acrosomal cap
b) Cytoplasmic droplet
c) Residual body
d) Sertoli vesicle
Explanation: Spermiogenesis involves elimination of excess cytoplasm, which is shed as residual bodies that are phagocytosed by Sertoli cells. Correct answer: c) Residual body.
10) Clinical case: A male presents with failure of fertilization despite normal sperm count and motility. Likely cause is defective:
a) Acrosome reaction
b) Mitochondria
c) DNA integrity
d) Sertoli cells
Explanation: Fertilization requires acrosome reaction, which releases enzymes to penetrate zona pellucida. Defective acrosome reaction leads to infertility. Correct answer: a) Acrosome reaction.
Topic: Fertilization and Implantation
Subtopic: Time of Implantation
Keyword Definitions:
• Fertilization: Fusion of sperm and ovum forming a zygote.
• Implantation: Process where blastocyst embeds into the endometrium.
• Blastocyst: Early embryonic stage consisting of inner cell mass and trophoblast.
• Endometrium: Uterine lining prepared for embryo implantation.
• Trophoblast: Outer cell layer of blastocyst involved in implantation.
• Luteal phase: Phase of menstrual cycle dominated by progesterone.
• hCG: Hormone secreted by trophoblast to support pregnancy.
• Zona pellucida: Glycoprotein covering around ovum preventing polyspermy.
• Morula: Solid ball of blastomeres formed before blastocyst stage.
• Decidua: Endometrium during pregnancy responding to implantation.
Lead Question - 2013
Implantation occurs after how many days of fertilization?
a) 3-5 days
b) 5-7 days
c) 7-9 days
d) > 14 days
Explanation: Implantation occurs typically on the 6th or 7th day after fertilization, when the blastocyst penetrates the endometrium. Fertilization occurs in the ampulla, cleavage forms morula, which develops into a blastocyst by day 4-5, and implantation begins by day 6-7. Correct answer: 7-9 days (c).
1) Cleavage of zygote results in formation of:
a) Blastocyst
b) Morula
c) Trophoblast
d) Decidua
Explanation: Cleavage is the rapid mitotic division of zygote forming a morula around day 3. The morula later develops into a blastocyst. This process is essential for implantation. Correct answer: Morula (b).
2) Clinical case: A woman with luteal phase defect may have recurrent implantation failure due to deficiency of:
a) Estrogen
b) Progesterone
c) LH
d) FSH
Explanation: Progesterone secreted by corpus luteum maintains endometrium. In luteal phase defect, low progesterone leads to implantation failure and infertility. Treatment involves progesterone supplementation. Correct answer: Progesterone (b).
3) Zona pellucida prevents:
a) Cleavage
b) Polyspermy
c) Fertilization
d) Ovulation
Explanation: Zona pellucida is a glycoprotein coat around the ovum that blocks multiple sperm entry after fertilization by cortical reaction. This ensures normal chromosomal complement. Correct answer: Polyspermy (b).
4) Clinical case: A patient with ectopic pregnancy most commonly shows implantation at:
a) Ampulla
b) Isthmus
c) Cervix
d) Ovary
Explanation: Most ectopic pregnancies occur in the fallopian tube, especially the ampullary region. This abnormal implantation is life-threatening and requires urgent management. Correct answer: Ampulla (a).
5) Hormone responsible for endometrial preparation for implantation:
a) Progesterone
b) Estrogen
c) LH
d) Prolactin
Explanation: Progesterone secreted by corpus luteum converts proliferative endometrium into secretory endometrium, suitable for implantation. Estrogen only stimulates proliferation but progesterone maintains pregnancy. Correct answer: Progesterone (a).
6) Clinical case: A woman develops vaginal bleeding 8 days post-ovulation. This corresponds to:
a) Follicular phase
b) Ovulatory phase
c) Implantation bleeding
d) Menstrual bleeding
Explanation: Implantation bleeding may occur around 7–9 days after ovulation, due to invasion of blastocyst into endometrium and disruption of blood vessels. Correct answer: Implantation bleeding (c).
7) Trophoblast differentiates into:
a) Cytotrophoblast and syncytiotrophoblast
b) Epiblast and hypoblast
c) Morula and blastocyst
d) Decidua and chorion
Explanation: Trophoblast cells of blastocyst differentiate into cytotrophoblast and syncytiotrophoblast, which play roles in implantation and hormone secretion (hCG). Correct answer: Cytotrophoblast and syncytiotrophoblast (a).
8) Clinical case: hCG is detected in maternal blood earliest by:
a) 1 day post-fertilization
b) 4 days post-fertilization
c) 8-9 days post-fertilization
d) 20 days post-fertilization
Explanation: Syncytiotrophoblast secretes hCG once implantation begins, which can be detected in maternal blood about 8-9 days after fertilization. Correct answer: 8-9 days (c).
9) The decidua basalis contributes to formation of:
a) Chorionic villi
b) Placenta
c) Amnion
d) Yolk sac
Explanation: The decidua basalis is the part of maternal endometrium directly beneath the implanted blastocyst. It fuses with chorionic villi to form the placenta. Correct answer: Placenta (b).
10) Clinical case: A woman on day 21 of her menstrual cycle shows endometrium with coiled glands and glycogen. This indicates:
a) Follicular phase
b) Proliferative phase
c) Secretory phase
d) Menstrual phase
Explanation: On day 21, progesterone action makes the endometrium secretory, preparing for implantation. Glands are coiled and glycogen-rich. Correct answer: Secretory phase (c).
Chapter: Reproductive Physiology
Topic: Breast Development
Subtopic: Hormonal Control of Mammary Glands
Keyword Definitions:
• Lactiferous ducts – Channels that carry milk from lobules to the nipple.
• Estrogen – Hormone responsible for ductal proliferation in breast tissue.
• Progesterone – Promotes lobuloalveolar development of mammary glands.
• Prolactin – Hormone stimulating milk secretion postpartum.
• Oxytocin – Causes milk ejection via myoepithelial contraction.
• LH & FSH – Pituitary gonadotropins regulating ovarian hormones that influence breast growth.
Lead Question - 2013
In breast lactiferous ducts are formed under the influence of which hormone?
a) Estrogen
b) Progesterone
c) LH
d) FSH
Explanation: Estrogen is the key hormone responsible for ductal growth and elongation of lactiferous ducts in the breast, particularly during puberty. Progesterone complements by stimulating alveolar and lobular development. LH and FSH regulate ovarian hormones indirectly. Therefore, the correct answer is Estrogen (option a).
1) Which hormone is mainly responsible for alveolar development of the breast?
a) Estrogen
b) Progesterone
c) Prolactin
d) Oxytocin
Explanation: Progesterone stimulates the lobuloalveolar system, preparing the breast for lactation. Estrogen primarily drives ductal development. Thus, alveolar development is under the influence of Progesterone (option b).
2) A lactating mother is unable to eject milk despite adequate production. Which hormone is deficient?
a) Estrogen
b) Progesterone
c) Oxytocin
d) Prolactin
Explanation: Oxytocin stimulates contraction of myoepithelial cells around alveoli, leading to milk ejection. Prolactin maintains milk production. Failure of ejection indicates Oxytocin deficiency (option c).
3) Which hormone maintains milk production during lactation?
a) Estrogen
b) Prolactin
c) Oxytocin
d) Progesterone
Explanation: Prolactin from the anterior pituitary maintains continuous milk synthesis postpartum. Oxytocin facilitates let-down, not production. Thus, the correct answer is Prolactin (option b).
4) A woman develops galactorrhea with amenorrhea. Likely hormone excess?
a) Prolactin
b) Estrogen
c) Progesterone
d) Oxytocin
Explanation: Galactorrhea with menstrual disturbances is classic of hyperprolactinemia, often due to pituitary adenoma. Thus, the correct answer is Prolactin (option a).
5) Which hormone inhibits lactation during pregnancy?
a) Estrogen and Progesterone
b) Prolactin
c) Oxytocin
d) Cortisol
Explanation: High estrogen and progesterone levels in pregnancy inhibit milk secretion despite high prolactin. After delivery, their fall allows prolactin to initiate lactation. Thus, the correct answer is Estrogen and Progesterone (option a).
6) A woman 3 days postpartum has engorged breasts with no milk secretion. Which hormone is insufficient?
a) Prolactin
b) Oxytocin
c) Estrogen
d) Progesterone
Explanation: Prolactin is required for milk synthesis in alveolar cells. Its deficiency leads to poor secretion despite breast engorgement. Thus, the correct answer is Prolactin (option a).
7) Which hormone increases during suckling to facilitate milk ejection?
a) Estrogen
b) Progesterone
c) Prolactin
d) Oxytocin
Explanation: Suckling reflex stimulates hypothalamus to release oxytocin from the posterior pituitary, which contracts myoepithelial cells for milk let-down. Hence, the answer is Oxytocin (option d).
8) A non-lactating woman with bilateral galactorrhea and visual disturbance likely has?
a) Hypothyroidism
b) Pituitary prolactinoma
c) Cushing’s syndrome
d) Addison’s disease
Explanation: Visual disturbances with galactorrhea suggest a pituitary mass compressing optic chiasma, most often prolactinoma. Thus, the correct answer is Pituitary prolactinoma (option b).
9) Which hormone is essential for breast ductal proliferation during puberty?
a) Estrogen
b) Progesterone
c) LH
d) FSH
Explanation: Estrogen stimulates ductal proliferation in breast tissue during puberty. Progesterone complements with alveolar development later. Thus, the answer is Estrogen (option a).
10) In lactation, which hormone prevents ovulation by suppressing GnRH?
a) Estrogen
b) Prolactin
c) Oxytocin
d) Progesterone
Explanation: Prolactin inhibits hypothalamic GnRH release, suppressing LH and FSH secretion, thereby preventing ovulation during lactation (lactational amenorrhea). Thus, the correct answer is Prolactin (option b).
11) A mother with Sheehan’s syndrome fails to lactate postpartum. Which hormone deficiency is responsible?
a) Estrogen
b) Prolactin
c) Oxytocin
d) Progesterone
Explanation: Sheehan’s syndrome is postpartum pituitary necrosis leading to loss of anterior pituitary hormones. Absence of prolactin prevents initiation of lactation. Thus, the correct answer is Prolactin (option b).
Topic: Male Reproductive Hormones
Subtopic: Testosterone Regulation
Keyword Definitions:
• Testosterone – Primary male sex hormone produced by Leydig cells.
• FSH – Follicle-stimulating hormone, regulates spermatogenesis.
• LH – Luteinizing hormone, stimulates Leydig cells to produce testosterone.
• Hypoandrogenism – Deficiency of androgens leading to reduced male sexual function.
• Spermatogenesis – Process of sperm cell development in seminiferous tubules.
• Negative Feedback – Mechanism by which testosterone suppresses LH/FSH release.
Lead Question - 2013
After injecting testosterone in a hypoandrogenic male, which of the following occurs ?
a) Decreased FSH secretion
b) Decreased LH secretion
c) Increased spermatogenesis
d) None of the above
Explanation: Testosterone supplementation exerts negative feedback on the hypothalamic-pituitary axis. This reduces LH secretion significantly as Leydig cells no longer need stimulation. FSH reduction may also occur, but LH suppression is most prominent. Thus, the correct answer is Decreased LH secretion (option b).
1) Which enzyme converts testosterone to dihydrotestosterone (DHT)?
a) Aromatase
b) 5-alpha reductase
c) 17-beta hydroxylase
d) 21-hydroxylase
Explanation: Testosterone is converted to dihydrotestosterone (DHT) by the enzyme 5-alpha reductase. DHT is a more potent androgen responsible for external genitalia development and male pattern baldness. Thus, the correct answer is 5-alpha reductase (option b).
2) A 25-year-old male has infertility with low sperm count and high FSH, normal testosterone. Which is most likely?
a) Klinefelter syndrome
b) Sertoli cell-only syndrome
c) Hypogonadotropic hypogonadism
d) Leydig cell tumor
Explanation: Elevated FSH with normal testosterone suggests defective Sertoli cell function while Leydig cells are intact. This is typical of Sertoli cell-only syndrome, leading to infertility despite normal androgen levels. Thus, the correct answer is Sertoli cell-only syndrome (option b).
3) Which hormone stimulates Leydig cells to produce testosterone?
a) FSH
b) LH
c) Prolactin
d) ACTH
Explanation: Luteinizing hormone (LH), secreted from the anterior pituitary, binds to receptors on Leydig cells of testes and stimulates testosterone production. FSH instead stimulates Sertoli cells. Therefore, the correct answer is LH (option b).
4) A 40-year-old man on long-term anabolic steroids develops testicular atrophy. Why?
a) Increased FSH
b) Increased LH
c) Suppression of gonadotropins
d) Overproduction of GnRH
Explanation: Exogenous anabolic steroids increase testosterone levels, which suppress LH and FSH through negative feedback. This suppression reduces testicular stimulation, causing atrophy. Thus, the correct answer is Suppression of gonadotropins (option c).
5) In males, inhibin is secreted by:
a) Leydig cells
b) Sertoli cells
c) Spermatogonia
d) Prostate
Explanation: Sertoli cells within the seminiferous tubules secrete inhibin, which provides negative feedback specifically on FSH secretion. Leydig cells secrete testosterone instead. Hence, the correct answer is Sertoli cells (option b).
6) A 19-year-old boy presents with gynecomastia and small firm testes. Karyotype: 47,XXY. Which is true?
a) Low testosterone, high FSH, high LH
b) High testosterone, low FSH
c) High testosterone, high LH
d) Low FSH, low LH
Explanation: Klinefelter syndrome features primary testicular failure, causing low testosterone and loss of negative feedback. Consequently, FSH and LH are elevated. Thus, the correct answer is Low testosterone, high FSH, high LH (option a).
7) Which of the following is NOT an effect of testosterone?
a) Growth of facial hair
b) Increased muscle mass
c) Development of breast tissue
d) Increased libido
Explanation: Testosterone is responsible for male secondary sexual characteristics such as facial hair, muscle mass, and libido. Gynecomastia or breast tissue development is not a direct effect; it may occur due to aromatization of testosterone to estrogen. Thus, the correct answer is Development of breast tissue (option c).
8) A 28-year-old man presents with infertility. Labs: low LH, low FSH, low testosterone. Diagnosis?
a) Primary hypogonadism
b) Secondary hypogonadism
c) Androgen resistance
d) Sertoli cell failure
Explanation: Simultaneous low LH, FSH, and testosterone indicate pituitary or hypothalamic dysfunction, i.e., secondary hypogonadism. Primary testicular failure would cause elevated gonadotropins. Hence, the correct answer is Secondary hypogonadism (option b).
9) Testosterone is mainly metabolized in:
a) Kidneys
b) Liver
c) Adrenal glands
d) Testes
Explanation: Testosterone is primarily metabolized in the liver, where it is converted into inactive metabolites and then excreted in urine. Hence, the correct answer is Liver (option b).
10) A 16-year-old boy has delayed puberty, low testosterone, anosmia. Which syndrome is suspected?
a) Turner syndrome
b) Kallmann syndrome
c) Androgen insensitivity syndrome
d) Noonan syndrome
Explanation: Kallmann syndrome is characterized by hypogonadotropic hypogonadism with anosmia due to failure of GnRH neurons to migrate. This causes low testosterone and absent puberty. Thus, the correct answer is Kallmann syndrome (option b).
11) Which hormone stimulates spermatogenesis directly via Sertoli cells?
a) FSH
b) LH
c) Testosterone
d) Prolactin
Explanation: FSH acts directly on Sertoli cells to promote spermatogenesis, whereas testosterone from Leydig cells provides paracrine support. Thus, the correct answer is FSH (option a).
Topic: Reproductive Hormones
Subtopic: Androgens
Keyword Definitions:
• Androgens: Male sex hormones responsible for development of male traits.
• Testosterone: Primary androgen secreted by Leydig cells of testes.
• Dihydrotestosterone (DHT): Potent androgen derived from testosterone by 5α-reductase.
• Androstenedione: Weak androgen, precursor for testosterone and estrogen.
• 17α-hydroxyprogesterone: Steroid intermediate in cortisol synthesis, not an androgen.
• Leydig cells: Testicular cells secreting testosterone.
• 5α-reductase: Enzyme converting testosterone into DHT.
Lead Question - 2013
All are androgens except?
a) Testosterone
b) Dihydrotestosterone
c) Androstenedione
d) 17a-hydroxyprogesterone
Explanation: Testosterone, DHT, and androstenedione are androgens that regulate male sexual development. 17α-hydroxyprogesterone, however, is a steroid intermediate in glucocorticoid synthesis and not an androgen. Therefore, the correct answer is 17α-hydroxyprogesterone.
1) Which enzyme converts testosterone to dihydrotestosterone?
a) Aromatase
b) 5α-reductase
c) 17β-HSD
d) Desmolase
Explanation: Testosterone is converted into potent dihydrotestosterone by 5α-reductase enzyme, particularly in prostate, skin, and hair follicles. Inhibition of this enzyme is used in benign prostatic hyperplasia treatment. Correct answer is 5α-reductase.
2) A 28-year-old male presents with infertility. Hormonal analysis shows low testosterone and high LH. Likely pathology?
a) Pituitary adenoma
b) Leydig cell failure
c) 21-hydroxylase deficiency
d) Klinefelter syndrome
Explanation: High LH with low testosterone indicates testicular (Leydig cell) failure, leading to hypogonadism and infertility. This is a form of primary hypogonadism. Correct answer is Leydig cell failure.
3) Main source of androstenedione in females?
a) Adrenal cortex
b) Theca cells of ovary
c) Granulosa cells
d) Corpus luteum
Explanation: Androstenedione in females is mainly produced by adrenal cortex and ovarian theca cells. It acts as a precursor for estrogen synthesis. Correct answer is Adrenal cortex.
4) A 6-year-old boy shows premature pubic hair and acne. Labs: high DHEA-S, normal testosterone. Diagnosis?
a) Central precocious puberty
b) Adrenal tumor
c) Leydig cell tumor
d) Pituitary adenoma
Explanation: High DHEA-S with normal testosterone suggests adrenal source androgen excess, most consistent with adrenal tumor or hyperplasia. Correct answer is Adrenal tumor.
5) Which androgen is most potent at androgen receptor?
a) Testosterone
b) DHEA
c) DHT
d) Androstenedione
Explanation: Dihydrotestosterone (DHT) binds more strongly to androgen receptors than testosterone or androstenedione. It mediates prostate growth, hair pattern, and external genitalia development. Correct answer is DHT.
6) A 25-year-old female with hirsutism has raised 17α-hydroxyprogesterone. Most likely disorder?
a) Polycystic ovary syndrome
b) Androgen-secreting tumor
c) Congenital adrenal hyperplasia
d) Ovarian failure
Explanation: High 17α-hydroxyprogesterone indicates 21-hydroxylase deficiency, a form of congenital adrenal hyperplasia, leading to androgen excess and hirsutism. Correct answer is Congenital adrenal hyperplasia.
7) Which androgen is secreted by adrenal glands?
a) Testosterone
b) DHEA
c) DHT
d) None
Explanation: Adrenal glands secrete weak androgens like DHEA and androstenedione, which can be converted to testosterone in peripheral tissues. Correct answer is DHEA.
8) A 30-year-old man has prostate cancer. Which drug reduces DHT levels?
a) Flutamide
b) Finasteride
c) Ketoconazole
d) Tamoxifen
Explanation: Finasteride inhibits 5α-reductase, blocking conversion of testosterone to DHT, thereby reducing prostate growth. It is used in prostate cancer and BPH. Correct answer is Finasteride.
9) Which androgen is an intermediate in estrogen synthesis?
a) Testosterone
b) DHT
c) Androstenedione
d) DHEA-S
Explanation: Androstenedione is converted to estrone by aromatase, serving as an intermediate in estrogen synthesis. Correct answer is Androstenedione.
10) A newborn girl has ambiguous genitalia. Labs: High 17α-hydroxyprogesterone. Likely diagnosis?
a) Turner syndrome
b) Congenital adrenal hyperplasia
c) PCOS
d) Androgen insensitivity
Explanation: Ambiguous genitalia in a newborn with high 17α-hydroxyprogesterone strongly suggests 21-hydroxylase deficiency, a type of congenital adrenal hyperplasia. Correct answer is Congenital adrenal hyperplasia.
Chapter: Reproductive Physiology
Topic: Oogenesis
Subtopic: Meiotic Arrest in Oocytes
Keyword Definitions:
• Oogenesis: Process of female gamete formation.
• Primary oocyte: Oocyte arrested in prophase I of meiosis.
• Diplotene stage: Substage of prophase I where oocytes arrest until puberty.
• Meiosis I: First meiotic division producing secondary oocyte and polar body.
• Metaphase II: Arrest stage of secondary oocyte until fertilization.
• Pachytene stage: Stage of crossing over in prophase I.
Lead Question - 2013
After first meiotic division, the primary oocyte remains arrested in?
a) Diplotene stage
b) Pachytene stage
c) Metaphase
d) Telophase
Explanation: Primary oocytes initiate meiosis during fetal life but arrest in prophase I at the diplotene stage until puberty. Secondary oocyte then arrests at metaphase II until fertilization. Hence, the correct answer is Diplotene stage, the site of first meiotic arrest.
1) Secondary oocyte is arrested at?
a) Diplotene
b) Metaphase I
c) Metaphase II
d) Telophase II
Explanation: After ovulation, the secondary oocyte proceeds to metaphase II, where it arrests until fertilization occurs. This ensures normal chromosomal segregation only after sperm entry. Correct answer is Metaphase II.
2) A 20-year-old woman undergoes ovulation induction. At ovulation, oocyte is arrested in?
a) Prophase I
b) Metaphase I
c) Metaphase II
d) Telophase I
Explanation: At ovulation, the oocyte is a secondary oocyte, arrested in metaphase II. This arrest persists until fertilization by sperm. Correct answer is Metaphase II.
3) Which hormone triggers completion of meiosis I in oocyte?
a) Estrogen
b) LH surge
c) Progesterone
d) FSH
Explanation: The LH surge triggers resumption of meiosis I in the primary oocyte, leading to its completion and formation of secondary oocyte and first polar body. Thus, the correct answer is LH surge.
4) A 28-year-old infertile female shows multiple primary oocytes arrested in diplotene. Likely diagnosis?
a) PCOS
b) Ovarian dysgenesis
c) Premature ovarian failure
d) Aromatase deficiency
Explanation: Arrest of primary oocytes in diplotene persisting without progression is seen in premature ovarian failure or dysgenesis, leading to infertility. Correct answer is Premature ovarian failure.
5) First polar body is extruded at?
a) Before ovulation
b) During ovulation
c) After fertilization
d) During puberty onset
Explanation: The first polar body is extruded at ovulation after meiosis I is completed under the influence of LH surge. Thus, correct answer is During ovulation.
6) A 24-year-old woman presents with infertility. Genetic analysis shows nondisjunction during meiosis I in oocyte. Possible outcome?
a) Turner syndrome
b) Klinefelter syndrome
c) Down syndrome
d) All of the above
Explanation: Nondisjunction during meiosis I of oocyte may lead to aneuploidy, resulting in disorders like Down, Turner, or Klinefelter syndromes. Thus, the correct answer is All of the above.
7) Diplotene arrest in oocytes is maintained by?
a) Estrogen
b) LH
c) Oocyte maturation inhibitor (OMI)
d) Progesterone
Explanation: OMI, secreted by granulosa cells, maintains diplotene arrest in primary oocytes until puberty. LH surge removes this inhibition. Correct answer is Oocyte maturation inhibitor.
8) A 30-year-old patient with infertility is given hCG injection. This mimics which natural hormone action on oocyte maturation?
a) FSH
b) LH
c) Progesterone
d) Prolactin
Explanation: hCG mimics LH surge, inducing resumption of meiosis I and ovulation. Thus, the correct answer is LH.
9) Fertilization completes which phase of meiosis in oocyte?
a) Meiosis I
b) Metaphase I
c) Meiosis II
d) Telophase I
Explanation: Fertilization triggers the completion of meiosis II in the secondary oocyte, leading to formation of a mature ovum and second polar body. Correct answer is Meiosis II.
10) A newborn girl has oocytes arrested in diplotene stage. This condition is?
a) Abnormal
b) Normal physiology
c) PCOS
d) Premature ovarian failure
Explanation: All primary oocytes in a newborn female are normally arrested in diplotene stage of prophase I until puberty. Thus, this is Normal physiology.
Chapter: Endocrinology
Topic: Steroidogenesis
Subtopic: Aromatase Enzyme Deficiency
Keyword Definitions:
• Aromatase: Enzyme that converts androgens to estrogens.
• Estrogen: Female sex hormone essential for reproduction.
• Testosterone: Primary male androgen, precursor for estrogen.
• Steroidogenesis: Hormone production from cholesterol.
• Cortisol: Stress hormone from adrenal cortex.
• Mineralocorticoids: Adrenal hormones regulating sodium and water balance.
Lead Question - 2013
Deficiency of enzyme aromatase leads to deficiency of which hormone?
a) Cortisol
b) Estrogen
c) Testosteron
d) Mineral corticoids
Explanation: Aromatase converts androgens like testosterone and androstenedione into estrogens. Deficiency results in impaired estrogen production, leading to virilization and infertility in females. Cortisol and mineralocorticoids are unaffected. Thus, the correct answer is Estrogen, the hormone directly deficient in aromatase deficiency.
1) Aromatase is located mainly in?
a) Ovary and placenta
b) Adrenal medulla
c) Liver
d) Kidney
Explanation: Aromatase is highly expressed in ovaries and placenta, converting androgens into estrogens during reproductive years and pregnancy. This ensures sufficient estrogen levels for female physiology. Hence, the correct answer is Ovary and placenta.
2) A 20-year-old female presents with primary amenorrhea, tall stature, and virilization. Likely cause?
a) Estrogen deficiency due to aromatase defect
b) Cortisol deficiency
c) Progesterone deficiency
d) Growth hormone excess
Explanation: Aromatase deficiency prevents conversion of testosterone to estrogen, causing virilization, amenorrhea, and tall stature due to delayed epiphyseal closure. Thus, the correct answer is Estrogen deficiency due to aromatase defect.
3) Which gene codes for aromatase enzyme?
a) CYP11B1
b) CYP17A1
c) CYP19A1
d) CYP21A2
Explanation: The aromatase enzyme is encoded by the CYP19A1 gene, located on chromosome 15. Mutations in this gene result in impaired estrogen synthesis, leading to aromatase deficiency syndromes. Correct answer is CYP19A1.
4) A pregnant woman with aromatase deficiency presents with virilization. Cause?
a) Fetal androgen excess
b) Maternal thyroid defect
c) Excess prolactin
d) High cortisol
Explanation: In aromatase deficiency, fetal androgens cannot be converted into estrogens, leading to maternal virilization during pregnancy. Hence, the correct answer is Fetal androgen excess.
5) Which of the following hormones is increased in aromatase deficiency?
a) Testosterone
b) Estrogen
c) Cortisol
d) Aldosterone
Explanation: In aromatase deficiency, androgens like testosterone accumulate because they are not converted into estrogens. Therefore, androgen levels are high while estrogen is low. Correct answer is Testosterone.
6) A 22-year-old female with aromatase deficiency presents with osteoporosis. Cause?
a) Cortisol excess
b) Estrogen deficiency
c) Progesterone deficiency
d) GH deficiency
Explanation: Estrogen is essential for bone mineralization and growth plate closure. Its deficiency in aromatase defect leads to osteoporosis and tall stature due to delayed epiphyseal fusion. Thus, the correct answer is Estrogen deficiency.
7) In males, aromatase is important for?
a) Spermatogenesis
b) Conversion of testosterone to estrogen
c) Testosterone secretion
d) LH production
Explanation: In males, aromatase converts testosterone to estrogen, which is vital for spermatogenesis and bone health. Hence, the correct answer is Conversion of testosterone to estrogen.
8) A male with aromatase deficiency is likely to show?
a) Early epiphyseal closure
b) Tall stature with osteoporosis
c) Short stature with obesity
d) Hypothyroidism
Explanation: Without estrogen, epiphyseal plates do not close, leading to tall stature and reduced bone density. Thus, the correct answer is Tall stature with osteoporosis.
9) Which laboratory finding supports aromatase deficiency?
a) Low estrogen, high androgen
b) Low cortisol, high estrogen
c) High progesterone, low testosterone
d) High mineralocorticoids
Explanation: Aromatase deficiency is confirmed by low estrogen with elevated androgens. Cortisol and mineralocorticoids remain normal. Hence, the correct answer is Low estrogen, high androgen.
10) Treatment of aromatase deficiency in females involves?
a) Cortisol supplementation
b) Estrogen replacement therapy
c) Progesterone only
d) Testosterone injections
Explanation: Estrogen replacement therapy is given to correct estrogen deficiency, prevent osteoporosis, and support normal secondary sexual characteristics. Thus, the correct answer is Estrogen replacement therapy.
Topic: Fertilization
Subtopic: Sperm Viability
Keyword Definitions:
• Sperm viability: Duration sperm remain alive and functional in female tract.
• Capacitation: Biochemical changes in sperm enabling fertilization.
• Acrosome reaction: Release of enzymes to penetrate ovum.
• Cervical mucus: Secretion that supports or hinders sperm survival.
• Fertilization window: Time during which conception is possible.
Lead Question - 2013
Human sperm remains fertile for how many hours in a female genital tract?
a) 6-8 hrs
b) 12-24 hrs
c) 24-48 hrs
d) 72-96 hrs
Explanation: After ejaculation, human sperm can survive for 48 to 72 hours, but optimal fertilizing capacity is usually within 24 to 48 hours. This viability depends on cervical mucus and female tract conditions. Thus, the correct answer is 24-48 hrs, representing peak fertilizing potential.
1) What is the minimum time required for sperm capacitation?
a) 1 hour
b) 4-6 hours
c) 12 hours
d) 24 hours
Explanation: Capacitation occurs in the female genital tract, taking about 4 to 6 hours. This process prepares sperm for acrosome reaction and fertilization. It does not occur immediately after ejaculation. Therefore, the correct answer is 4-6 hours, critical for fertilization.
2) A 28-year-old woman with regular cycles wants to conceive. Best time for intercourse?
a) Day 1-3
b) Day 5-9
c) Day 12-16
d) Day 20-25
Explanation: Fertile period is around ovulation, typically between day 12-16 in a 28-day cycle. Intercourse during this period provides maximum chance of conception, as sperm remain viable and ovum survives 12-24 hours. Hence, the correct answer is Day 12-16.
3) Which female structure stores sperm temporarily?
a) Fallopian tube
b) Cervix
c) Ovary
d) Uterus
Explanation: Cervical mucus and cervical crypts serve as reservoirs for sperm, allowing gradual release into the uterus and tubes. This mechanism prolongs fertility and enhances conception chances. Thus, the correct answer is Cervix, which helps sperm survival.
4) A 32-year-old woman with infertility is found to have thick cervical mucus. Likely effect?
a) Increased sperm motility
b) Reduced sperm survival
c) Early ovulation
d) Increased implantation
Explanation: Thick, hostile cervical mucus impedes sperm entry and reduces their survival, often causing infertility. Normally, estrogen makes mucus watery to support sperm passage. Hence, the correct answer is Reduced sperm survival.
5) How long does the ovum remain viable for fertilization?
a) 6-8 hours
b) 12-24 hours
c) 48-72 hours
d) 96 hours
Explanation: The ovum remains viable for only 12 to 24 hours after ovulation. Unlike sperm, eggs have a shorter fertile lifespan. Fertilization must occur within this window for successful conception. Therefore, the correct answer is 12-24 hours.
6) A couple presents with infertility. Semen analysis shows normal sperm count but no pregnancy after 1 year. Possible reason?
a) Short sperm viability
b) Anovulation
c) High testosterone
d) Low LH
Explanation: If sperm parameters are normal, female factors like anovulation are common causes of infertility. Without ovulation, fertilization cannot occur despite viable sperm. Hence, the correct answer is Anovulation.
7) Sperm undergo acrosome reaction in which part of female tract?
a) Cervix
b) Vagina
c) Ampulla of fallopian tube
d) Uterus
Explanation: Acrosome reaction occurs in the ampullary region of the fallopian tube, where fertilization normally takes place. This ensures penetration of the zona pellucida. Therefore, the correct answer is Ampulla of fallopian tube.
8) A 26-year-old woman uses barrier contraception. How does it prevent fertilization?
a) Prevents ovulation
b) Blocks sperm entry
c) Alters endometrium
d) Suppresses LH surge
Explanation: Barrier methods like condoms prevent fertilization by blocking sperm from entering the female genital tract. They do not affect ovulation or endometrium. Thus, the correct answer is Blocks sperm entry.
9) Sperm motility depends primarily on?
a) Flagellar activity
b) Mitochondrial ATP
c) Calcium influx
d) All of the above
Explanation: Sperm motility is maintained by ATP from mitochondria, flagellar action, and calcium influx regulating motility. All mechanisms contribute synergistically. Thus, the correct answer is All of the above.
10) A patient with hyperprolactinemia presents with infertility. Cause?
a) Reduced sperm viability
b) Failure of ovulation
c) Uterine abnormalities
d) Tubal block
Explanation: Hyperprolactinemia suppresses GnRH, reducing LH and FSH, leading to anovulation. This prevents conception despite normal sperm viability. Hence, the correct answer is Failure of ovulation, commonly caused by prolactinomas or antipsychotic drugs.
Topic: Reproductive Physiology
Subtopic: Ovulation and Hormonal Regulation
Keyword Definitions:
• LH Surge: Rapid rise of luteinizing hormone before ovulation.
• Estrogen: Ovarian hormone responsible for endometrial proliferation.
• Progesterone: Hormone secreted mainly after ovulation by corpus luteum.
• Ovulation: Release of mature ovum triggered by LH surge.
• Follicle: Ovarian structure containing developing oocyte.
• Corpus Luteum: Post-ovulatory structure secreting progesterone.
• FSH: Follicle stimulating hormone aiding follicular maturation.
• Menstrual Cycle: Regular cyclic changes preparing for pregnancy.
• Hypothalamus: Releases GnRH to regulate LH and FSH.
• GnRH: Gonadotropin releasing hormone, triggers pituitary gonadotropins.
Lead Question - 2013
LH surge is associated with?
a) Increased estrogen & decreased progesterone
b) Increased estrogen & increased progesterone
c) Decreased estrogen & increased progesterone
d) Decreased estrogen & increased progesterone
Explanation: The LH surge occurs due to sustained high estrogen levels from the dominant follicle, which switches feedback from negative to positive on the hypothalamus and pituitary. Progesterone rises only after ovulation. Therefore, LH surge is associated with increased estrogen and decreased progesterone. The correct answer is a) Increased estrogen & decreased progesterone.
1) Which hormone is essential for maintaining luteal phase?
a) Estrogen
b) Progesterone
c) FSH
d) LH
Explanation: The luteal phase is maintained by progesterone, secreted by the corpus luteum under the influence of LH. Progesterone ensures endometrial receptivity and prepares for possible implantation. Without adequate progesterone, menstruation begins. Hence, the correct answer is b) Progesterone.
2) A woman presents with infertility. Her cycles are regular but without mid-cycle LH surge. What is most likely absent?
a) Follicular recruitment
b) Dominant follicle development
c) Corpus luteum formation
d) Estrogen production
Explanation: Absence of LH surge prevents ovulation, meaning no corpus luteum formation and no progesterone production. Estrogen may still be present from follicular phase. Thus, the most likely absent feature is c) Corpus luteum formation.
3) Which ovarian hormone has positive feedback on LH surge?
a) Estrogen
b) Progesterone
c) Inhibin
d) Activin
Explanation: Estrogen, when sustained at high levels, exerts a positive feedback effect on the hypothalamus and pituitary, resulting in the LH surge that triggers ovulation. Progesterone and inhibin act mainly with negative feedback. Hence, the correct answer is a) Estrogen.
4) A patient has mid-cycle pelvic pain (Mittelschmerz). Which hormonal event is most closely related?
a) LH surge
b) Decline in FSH
c) Rise in progesterone
d) Fall in estrogen
Explanation: Mittelschmerz, or mid-cycle pain, coincides with follicular rupture during ovulation. This event is triggered by the LH surge. Thus, the hormonal event most closely related is a) LH surge.
5) Which hormone prevents multiple ovulations during one cycle?
a) Estrogen
b) Progesterone
c) Inhibin
d) LH
Explanation: Progesterone from corpus luteum provides negative feedback on GnRH, LH, and FSH secretion, preventing additional ovulations within the same cycle. Thus, the correct answer is b) Progesterone.
6) A 28-year-old woman has irregular cycles. Serum shows no mid-cycle LH surge. What phase defect is present?
a) Luteal phase defect
b) Anovulatory cycle
c) Short follicular phase
d) Hyperprolactinemia
Explanation: Lack of mid-cycle LH surge indicates failure of ovulation, making the cycle anovulatory. Luteal phase cannot occur without ovulation. Thus, the correct answer is b) Anovulatory cycle.
7) Which structure produces progesterone after ovulation?
a) Theca interna
b) Granulosa lutein cells
c) Oocyte
d) Hypothalamus
Explanation: After ovulation, granulosa cells become granulosa lutein cells in the corpus luteum, which secrete progesterone. Theca interna produces androgens before ovulation. Thus, the correct answer is b) Granulosa lutein cells.
8) A woman on clomiphene therapy develops multiple follicles. What does clomiphene inhibit?
a) Estrogen receptors in hypothalamus
b) Progesterone receptors in uterus
c) LH receptors in ovary
d) FSH receptors in ovary
Explanation: Clomiphene is an anti-estrogen that blocks hypothalamic estrogen receptors, preventing negative feedback, increasing GnRH and stimulating FSH/LH release, leading to follicle growth. Hence, the correct answer is a) Estrogen receptors in hypothalamus.
9) Which event follows LH surge most directly?
a) Follicle rupture
b) Endometrial shedding
c) Decline in estrogen
d) Inhibin secretion
Explanation: The LH surge directly causes follicle rupture, leading to ovulation. Other events like endometrial shedding occur later if implantation does not happen. Thus, the correct answer is a) Follicle rupture.
10) A 30-year-old woman with PCOS has persistently high estrogen but no LH surge. What is the likely cause?
a) Absent GnRH pulses
b) Progesterone deficiency
c) Loss of estrogen positive feedback
d) Excess androgen production
Explanation: In PCOS, estrogen is present but fails to induce an LH surge due to altered GnRH pulsatility and high androgens. The key mechanism is loss of estrogen’s positive feedback effect. The correct answer is c) Loss of estrogen positive feedback.
Topic: Reproductive Physiology
Subtopic: Blood-Testis Barrier
Keyword Definitions:
• Blood-testis barrier: A physical barrier formed by tight junctions between Sertoli cells, preventing harmful substances from reaching developing sperm.
• Sertoli cells: Supporting cells of seminiferous tubules that provide structural and nutritional support to developing germ cells.
• Basal lamina: Thin extracellular layer forming the basement membrane of seminiferous tubules.
• Leydig cells: Interstitial cells of testis producing testosterone.
• Spermatogonia: Diploid germ cells located near basal lamina, precursors of spermatozoa.
• Tight junctions: Specialized intercellular connections that restrict paracellular movement of substances.
• Immunological privilege: Mechanism preventing immune attack on germ cells.
• Seminiferous tubules: Site of spermatogenesis within testes.
• Spermatogenesis: Process of sperm formation from spermatogonia.
• Androgen: Steroid hormones like testosterone produced by Leydig cells.
Lead Question - 2013
Blood tissue barrier in testis is formed by?
a) Basal lamina & interstitial cells
b) Adjacent sertoli cells with basal lamina
c) Basal lamina & spermatogonia
d) Basal lamina & leydig cells
Explanation: The blood-testis barrier is primarily formed by tight junctions between adjacent Sertoli cells, reinforced by the basal lamina. It separates basal and adluminal compartments, protecting developing germ cells from immune attack. Correct answer is b) Adjacent Sertoli cells with basal lamina. This barrier is vital for spermatogenesis.
1) Which cells secrete androgen-binding protein?
a) Leydig cells
b) Sertoli cells
c) Spermatogonia
d) Peritubular cells
Explanation: Sertoli cells produce androgen-binding protein under FSH influence. This protein maintains high testosterone concentration in seminiferous tubules, essential for spermatogenesis. Leydig cells produce testosterone, not ABP. Correct answer is b) Sertoli cells. Spermatogonia are germ cells, and peritubular cells provide contractile support.
2) Clinical: In mumps orchitis, which cells are primarily damaged?
a) Sertoli cells
b) Leydig cells
c) Spermatogonia
d) Basal lamina
Explanation: Mumps orchitis often damages Leydig cells, impairing testosterone production and leading to infertility. Sertoli cells may be affected secondarily, but primary pathology involves Leydig cells. Thus, correct answer is b) Leydig cells. Spermatogonia and basal lamina remain less affected initially.
3) Which hormone stimulates Leydig cells?
a) FSH
b) LH
c) Prolactin
d) Testosterone
Explanation: Luteinizing hormone (LH) stimulates Leydig cells to synthesize testosterone, essential for spermatogenesis and male secondary sexual characteristics. FSH acts on Sertoli cells, prolactin modulates LH effects, testosterone provides negative feedback. Correct answer is b) LH. This axis is key for reproductive physiology.
4) Clinical: A patient with low FSH has impaired spermatogenesis but normal testosterone. Which cells are affected?
a) Sertoli cells
b) Leydig cells
c) Spermatogonia
d) Peritubular cells
Explanation: FSH acts on Sertoli cells to support spermatogenesis, while LH regulates Leydig cells and testosterone. Low FSH impairs Sertoli function, disrupting germ cell maturation despite normal testosterone. Correct answer is a) Sertoli cells. This highlights the dual regulation of male fertility.
5) Which of the following forms the structural support in seminiferous tubules?
a) Spermatogonia
b) Sertoli cells
c) Leydig cells
d) Peritubular fibroblasts
Explanation: Sertoli cells act as “nurse cells” that provide structural support, nutrients, and paracrine signaling for germ cells in seminiferous tubules. Leydig cells lie outside tubules. Correct answer is b) Sertoli cells. Spermatogonia are developing germ cells, not supportive structures.
6) Clinical: A man has antibodies against his sperm. Which barrier failed?
a) Blood-testis barrier
b) Hemato-encephalic barrier
c) Placental barrier
d) Mucosal barrier
Explanation: The blood-testis barrier prevents sperm antigens from exposure to immune system. If it breaks, immune cells recognize sperm as foreign, causing infertility. Correct answer is a) Blood-testis barrier. Other barriers like placental or brain barriers are unrelated to spermatogenesis.
7) Which compartment houses spermatogonia?
a) Adluminal
b) Basal
c) Luminal
d) Interstitial
Explanation: Spermatogonia, the earliest germ cells, lie in the basal compartment of seminiferous tubules, near the basal lamina, outside the blood-testis barrier. As they mature, spermatocytes move into the adluminal compartment. Correct answer is b) Basal. This spatial separation is crucial for controlled germ cell development.
8) Clinical: A 25-year-old male presents with infertility and low inhibin levels. Which cells are defective?
a) Leydig cells
b) Sertoli cells
c) Spermatogonia
d) Basal lamina
Explanation: Sertoli cells secrete inhibin, which inhibits FSH. Low inhibin indicates Sertoli cell dysfunction, impairing spermatogenesis. Testosterone from Leydig cells may be normal. Correct answer is b) Sertoli cells. This emphasizes their dual role in germ cell support and hormonal regulation.
9) Testosterone diffuses into tubules through?
a) ABP binding
b) Exocytosis
c) Tight junctions
d) Osmosis
Explanation: Testosterone from Leydig cells enters seminiferous tubules bound to androgen-binding protein (ABP), maintaining high concentration for spermatogenesis. Thus, correct answer is a) ABP binding. It is not stored or secreted via exocytosis like peptides.
10) Clinical: Which condition disrupts spermatogenesis by breaking Sertoli cell junctions?
a) Chemotherapy
b) Diabetes
c) Hypertension
d) Hypothyroidism
Explanation: Chemotherapy drugs are toxic to rapidly dividing cells, including germ cells. They also disrupt Sertoli cell junctions forming the blood-testis barrier, causing infertility. Correct answer is a) Chemotherapy. Diabetes and hypertension affect vasculature, but do not directly disrupt Sertoli junctions.
Topic: Reproductive Physiology
Subtopic: Lactation Hormones
Keyword Definitions:
- Prolactin: Hormone from anterior pituitary stimulating milk production in mammary glands.
- Oxytocin: Posterior pituitary hormone causing milk ejection and uterine contraction.
- Galactopoiesis: Maintenance of milk production, mainly prolactin-mediated.
- Lactogenesis: Onset of milk secretion after childbirth.
- Relaxin: Hormone softening pelvic ligaments during pregnancy, not linked to lactation.
Lead Question - 2013
Primary hormone for secretion of milk ?
a) Oxytocin
b) Prolactin
c) Glucocorticoids
d) Relaxin
Answer and Explanation:
Correct answer is b) Prolactin. Prolactin from anterior pituitary is the primary hormone responsible for milk secretion and galactopoiesis. Oxytocin plays a role in milk ejection reflex but not production. Glucocorticoids support gland development, while relaxin affects pelvic ligaments but not lactation. Thus, prolactin is essential for milk secretion.
Guessed Questions for NEET PG:
1. Which hormone is essential for milk ejection reflex?
a) Prolactin
b) Oxytocin
c) Estrogen
d) Progesterone
Explanation: Correct answer is b) Oxytocin. Released from posterior pituitary, oxytocin stimulates contraction of myoepithelial cells around alveoli, pushing milk into ducts during suckling reflex.
2. Galactorrhea is caused by excess:
a) Estrogen
b) Progesterone
c) Prolactin
d) Oxytocin
Explanation: Correct answer is c) Prolactin. Hyperprolactinemia leads to galactorrhea, infertility, and menstrual disturbances. Commonly due to prolactinoma or dopamine antagonist drugs affecting prolactin regulation.
3. A lactating mother unable to eject milk despite normal production likely has deficiency of:
a) Prolactin
b) Oxytocin
c) Estrogen
d) Progesterone
Explanation: Correct answer is b) Oxytocin. Oxytocin deficiency or blocked reflex prevents milk ejection though prolactin maintains milk synthesis. Stress and anxiety also inhibit oxytocin release.
4. Which hormone inhibits milk secretion during pregnancy despite high prolactin levels?
a) Estrogen
b) Progesterone
c) Both estrogen and progesterone
d) Cortisol
Explanation: Correct answer is c) Both estrogen and progesterone. High levels block prolactin action on mammary alveoli. After delivery, their fall permits lactogenesis and milk secretion.
5. In Sheehan’s syndrome, failure of lactation occurs due to deficiency of:
a) Prolactin
b) Oxytocin
c) Growth hormone
d) ACTH
Explanation: Correct answer is a) Prolactin. Postpartum pituitary necrosis reduces anterior pituitary hormones, especially prolactin, leading to failure of lactation along with amenorrhea and hypothyroidism.
6. Which receptor mediates prolactin secretion regulation?
a) Dopamine D2 receptor
b) Serotonin 5-HT1 receptor
c) GABA receptor
d) Glucocorticoid receptor
Explanation: Correct answer is a) Dopamine D2 receptor. Dopamine inhibits prolactin secretion through D2 receptors. Dopamine antagonists increase prolactin causing galactorrhea and menstrual irregularities.
7. A mother with hypothyroidism presents with low milk secretion. Likely due to reduced:
a) Prolactin
b) TSH
c) TRH stimulation of prolactin
d) Oxytocin
Explanation: Correct answer is c) TRH stimulation of prolactin. Hypothyroidism causes reduced TRH, decreasing prolactin stimulation and impairing lactation. TRH normally enhances prolactin secretion in pituitary.
8. Which factor stimulates prolactin release during breastfeeding?
a) Stretch of cervix
b) Suckling reflex
c) Increased estrogen
d) Increased cortisol
Explanation: Correct answer is b) Suckling reflex. Nipple stimulation during suckling reduces hypothalamic dopamine inhibition, increasing prolactin and oxytocin release for milk production and ejection.
9. Which pituitary lobe secretes prolactin?
a) Posterior lobe
b) Intermediate lobe
c) Anterior lobe
d) All lobes
Explanation: Correct answer is c) Anterior lobe. Prolactin is secreted by lactotrophs of anterior pituitary. Posterior pituitary stores and releases oxytocin and vasopressin, not prolactin.
10. Bromocriptine suppresses lactation by:
a) Stimulating dopamine receptors
b) Blocking estrogen receptors
c) Inhibiting oxytocin receptors
d) Inhibiting progesterone receptors
Explanation: Correct answer is a) Stimulating dopamine receptors. Bromocriptine, a dopamine agonist, inhibits prolactin secretion via D2 receptors, suppressing milk production in hyperprolactinemia and unwanted lactation.
Topic: Peritoneal Cavity
Subtopic: Peritoneal Pouches in Female
Keyword Definitions:
Peritoneal cavity: Potential space between parietal and visceral peritoneum containing serous fluid.
Vesicouterine pouch: Peritoneal recess between urinary bladder and uterus.
Pouch of Douglas (Rectouterine pouch): Most dependent peritoneal recess between uterus and rectum in females.
Dependent part: Lowest part in standing position where fluid accumulates.
Clinical significance: Site for fluid collection in ascites or infections; accessible for culdocentesis.
Culdocentesis: Diagnostic procedure to aspirate fluid from pouch of Douglas.
Lead Question - 2013
Most dependent part of abdomen in standing position is ?
a) Vesicouterine pouch
b) Pouch of Douglas
c) Rectouterine pouch
d) b & c
Explanation: The most dependent part of the female peritoneal cavity in the standing position is the pouch of Douglas (also called the rectouterine pouch). Both terms describe the same anatomical space. This is clinically significant as fluid accumulates here during peritoneal infections or ascites. Therefore, correct answer is d) b & c.
Guessed Question 2
Which structure lies anterior to pouch of Douglas?
a) Rectum
b) Uterus
c) Bladder
d) Sigmoid colon
Explanation: The pouch of Douglas lies between the uterus and rectum, making the uterus anterior to it. This anatomical relation is important in clinical procedures like culdocentesis. Correct answer is b) Uterus.
Guessed Question 3
Clinical significance of pouch of Douglas?
a) Site of fluid accumulation
b) Site of tumor implantation
c) Accessible for culdocentesis
d) All of the above
Explanation: The pouch of Douglas is significant for accumulating fluid in peritoneal diseases, tumor implantation, and is accessible by culdocentesis to sample fluid for diagnosis. Correct answer is d) All of the above.
Guessed Question 4
Vesicouterine pouch lies between?
a) Bladder and uterus
b) Uterus and rectum
c) Bladder and rectum
d) Uterus and ovary
Explanation: The vesicouterine pouch is the peritoneal recess between the urinary bladder and the uterus in females. It is anterior to the uterus. Correct answer is a) Bladder and uterus.
Guessed Question 5
Why is pouch of Douglas the lowest point in females?
a) Due to body posture
b) Uterus placement
c) Peritoneal reflections
d) All of the above
Explanation: In standing position, anatomical arrangement and peritoneal reflections make the rectouterine pouch (pouch of Douglas) the lowest point, where fluid accumulates in pathological conditions. Correct answer is d) All of the above.
Guessed Question 6
Pouch of Douglas is clinically accessed via?
a) Laparoscopy
b) Culdocentesis
c) Paracentesis
d) Colonoscopy
Explanation: Culdocentesis is a clinical procedure where fluid is aspirated from the pouch of Douglas via the posterior vaginal fornix to diagnose infections or fluid accumulation. Correct answer is b) Culdocentesis.
Guessed Question 7
Which organ does NOT relate to pouch of Douglas?
a) Uterus
b) Bladder
c) Rectum
d) Small intestine
Explanation: The pouch of Douglas is bordered by uterus and rectum, with bladder more anteriorly. The small intestine does not specifically relate to this peritoneal recess. Correct answer is d) Small intestine.
Guessed Question 8
In males, equivalent of pouch of Douglas is?
a) Rectovesical pouch
b) Vesicouterine pouch
c) Pararectal pouch
d) None
Explanation: In males, the rectovesical pouch lies between the rectum and urinary bladder, analogous to the female pouch of Douglas. It serves as the most dependent peritoneal space in males. Correct answer is a) Rectovesical pouch.
Guessed Question 9
What does accumulation of pus in pouch of Douglas suggest?
a) Appendicitis
b) Pelvic inflammatory disease
c) Peritonitis
d) All of the above
Explanation: Pus accumulation in the pouch of Douglas typically suggests pelvic inflammatory disease, though other infections like appendicitis or peritonitis can also cause fluid collection there. Correct answer is d) All of the above.
Guessed Question 10
Pouch of Douglas is bounded by:
a) Posterior vaginal fornix
b) Uterus
c) Rectum
d) All of the above
Explanation: The pouch of Douglas is anatomically bounded by the posterior vaginal fornix, uterus anteriorly, and rectum posteriorly. This area is clinically important in gynecology. Correct answer is d) All of the above.
Chapter: Anatomy
Topic: Male Reproductive System
Subtopic: Vascular Supply of Penis
Keyword Definitions:
Helicine arteries: Small, coiled arteries in the corpora cavernosa of the penis responsible for erectile function by regulating blood flow.
Deep artery of penis: Main artery supplying corpora cavernosa, gives rise to helicine arteries.
External pudendal artery: Branch of femoral artery supplying skin of external genitalia, not directly giving rise to helicine arteries.
Femoral artery: Major artery of lower limb, not directly involved in penile vascularization.
Corpora cavernosa: Paired erectile tissues in penis filled by blood during erection.
Corpus spongiosum: Surrounds urethra, supplied by other arteries.
Erection mechanism: Helicine arteries dilate under parasympathetic stimulation, allowing blood into erectile tissue.
Lead Question - 2013
Helicine artery are branch of ?
a) Deep artery of penis
b) Femoral artery
c) External pudendal artery
d) None of the above
Explanation: Helicine arteries are branches of the deep artery of penis. These arteries are crucial for erectile function, as they supply the corpora cavernosa and regulate blood flow into the erectile tissue during sexual arousal. Therefore, correct answer is a) Deep artery of penis.
Guessed Question 2
Function of helicine arteries?
a) Supply corpus spongiosum
b) Supply corpora cavernosa
c) Drain venous blood
d) Supply skin of penis
Explanation: Helicine arteries are specialized branches of the deep artery of penis that supply the corpora cavernosa, regulating blood inflow during erection. Their coiled structure allows dilation. Correct answer is b) Supply corpora cavernosa.
Guessed Question 3
Which nerve controls helicine artery dilation?
a) Sympathetic
b) Parasympathetic
c) Somatic
d) None
Explanation: Parasympathetic stimulation causes helicine arteries to dilate, allowing increased blood flow into corpora cavernosa, leading to penile erection. Sympathetic stimulation causes constriction. Correct answer is b) Parasympathetic.
Guessed Question 4
Deep artery of penis is branch of?
a) Internal pudendal artery
b) External iliac artery
c) Femoral artery
d) Aorta
Explanation: The deep artery of penis is a branch of the internal pudendal artery, which supplies the penis, perineum, and adjacent structures. Correct answer is a) Internal pudendal artery.
Guessed Question 5
Clinical relevance of helicine artery dysfunction?
a) Erectile dysfunction
b) Priapism
c) Phimosis
d) Paraphimosis
Explanation: Impaired helicine artery function reduces blood inflow to corpora cavernosa, causing erectile dysfunction. Hyperfunction may lead to priapism. Correct answer is a) Erectile dysfunction.
Guessed Question 6
Which structure does NOT receive blood from helicine arteries?
a) Corpora cavernosa
b) Corpus spongiosum
c) Glans penis
d) Tunica albuginea
Explanation: Helicine arteries supply corpora cavernosa directly. Corpus spongiosum is supplied by different branches (e.g., artery of bulb of penis). Correct answer is b) Corpus spongiosum.
Guessed Question 7
Helicine arteries remain contracted in?
a) Flaccid state
b) Erect state
c) After ejaculation
d) None of the above
Explanation: In the flaccid state, helicine arteries remain contracted, limiting blood flow into corpora cavernosa. Upon arousal, they dilate. Correct answer is a) Flaccid state.
Guessed Question 8
Blockage of which artery can cause penile ischemia?
a) Deep artery of penis
b) External pudendal artery
c) Femoral artery
d) Internal thoracic artery
Explanation: Blockage of the deep artery of penis can result in insufficient blood flow to the corpora cavernosa, leading to ischemia and erectile dysfunction. Correct answer is a) Deep artery of penis.
Guessed Question 9
Which artery is not involved in penile blood supply?
a) Deep artery of penis
b) Dorsal artery of penis
c) Helicine artery
d) Coronary artery
Explanation: The coronary artery supplies the heart and has no role in penile blood supply. Other arteries supply various penile structures. Correct answer is d) Coronary artery.
Guessed Question 10
Helicine arteries' role during sexual arousal is:
a) Constrict
b) Remain unchanged
c) Dilate
d) Regress
Explanation: During sexual arousal, helicine arteries dilate under parasympathetic stimulation, allowing blood to fill the corpora cavernosa and produce erection. Correct answer is c) Dilate.
Chapter: Anatomy
Topic: Female Reproductive System
Subtopic: Cervix Structure
Keyword Definitions:
Cervix: Lower part of uterus connecting uterine cavity to vagina, composed of connective tissue and smooth muscle.
Connective tissue: Provides structural support, rich in collagen and elastin fibers, predominant in cervix.
Smooth muscle: Involuntary muscle fibers present in cervical stroma, responsible for contractility during labor.
Cervical stroma: Tissue matrix of cervix containing both connective tissue and smooth muscle, contributing to strength and elasticity.
Ratio connective tissue : smooth muscle: Determines cervical rigidity and ability to dilate during childbirth.
Clinical relevance: Understanding tissue ratio is essential in cervical surgeries, obstetrics, and assessment of cervical insufficiency.
Fornices: Recesses around cervix within vaginal canal, composed mainly of connective tissue.
Lead Question - 2013
Ratio of connective tissue : smooth muscle in cervix is ?
a) 2:1
b) 5:1
c) 8:1
d) None
Explanation: The cervical stroma is composed predominantly of connective tissue, which provides tensile strength, while smooth muscle is comparatively lesser. Histological studies show the ratio of connective tissue to smooth muscle in the cervix is approximately 5:1. Correct answer is b) 5:1.
Guessed Question 2
Which component predominates in cervical stroma?
a) Smooth muscle
b) Connective tissue
c) Epithelium
d) Cartilage
Explanation: Connective tissue is the major component of cervical stroma, providing tensile strength and elasticity. Smooth muscle is less abundant. Epithelium lines the canal but is not part of stroma. Correct answer is b) Connective tissue.
Guessed Question 3
Cervical smooth muscle is important for?
a) Contractions during labor
b) Hormone production
c) Structural support
d) Blood supply
Explanation: Smooth muscle in the cervix contributes to contractile function, allowing dilation during labor. It does not produce hormones or provide primary structural support. Correct answer is a) Contractions during labor.
Guessed Question 4
Which type of connective tissue is abundant in cervix?
a) Collagen
b) Elastic cartilage
c) Bone
d) Adipose tissue
Explanation: Collagen fibers are abundant in cervical connective tissue, providing tensile strength and rigidity, while elastin allows flexibility. Correct answer is a) Collagen.
Guessed Question 5
Cervical insufficiency is due to:
a) Weak connective tissue
b) Excess smooth muscle
c) Infection
d) Fibrosis
Explanation: Weak connective tissue in cervical stroma can lead to cervical insufficiency, resulting in premature dilation and pregnancy loss. Smooth muscle deficiency is not a primary cause. Correct answer is a) Weak connective tissue.
Guessed Question 6
During pregnancy, cervical connective tissue:
a) Becomes more rigid
b) Becomes softer and more elastic
c) Converts to smooth muscle
d) Does not change
Explanation: Hormonal changes during pregnancy cause remodeling of cervical connective tissue, increasing elasticity and allowing dilation at labor. Correct answer is b) Becomes softer and more elastic.
Guessed Question 7
Which layer forms the bulk of cervical wall?
a) Stroma
b) Epithelium
c) Mucosa
d) Serosa
Explanation: The cervical stroma forms the bulk of cervical wall, composed mainly of connective tissue with interspersed smooth muscle. Epithelium lines the canal. Correct answer is a) Stroma.
Guessed Question 8
Clinical importance of connective tissue in cervix?
a) Determines cervical strength
b) Enables dilation
c) Supports uterus
d) All of the above
Explanation: Connective tissue in cervix provides structural integrity, allows remodeling and dilation during labor, and supports uterine position. Correct answer is d) All of the above.
Guessed Question 9
Major histological change of cervix during labor:
a) Connective tissue remodeling
b) Smooth muscle hypertrophy
c) Epithelium proliferation
d) Calcification
Explanation: Cervical connective tissue undergoes remodeling under hormonal influence (collagen breakdown, increased water content), enabling dilation. Correct answer is a) Connective tissue remodeling.
Guessed Question 10
Ratio of smooth muscle : connective tissue in cervix?
a) 1:5
b) 1:2
c) 2:1
d) 5:1
Explanation: Since connective tissue predominates with a ratio of 5:1, smooth muscle : connective tissue ratio is 1:5. Correct answer is a) 1:5.
Guessed Question 11
Cervical collagen provides:
a) Tensile strength
b) Contractility
c) Hormonal function
d) Vascular supply
Explanation: Collagen fibers in cervical stroma provide tensile strength and rigidity, essential for structural support of uterus and controlled dilation during labor. Correct answer is a) Tensile strength.
Chapter: Anatomy
Topic: Male Reproductive System
Subtopic: Spermatic Cord & Fasciae
Keyword Definitions:
Internal spermatic fascia: Thin fascia surrounding spermatic cord and testis, derived from transversalis fascia, lies deep to cremasteric fascia.
Cremasteric fascia: Fascial layer derived from internal oblique muscle, contains cremasteric muscle fibers, elevates testis.
External spermatic fascia: Derived from external oblique aponeurosis, superficial to cremasteric fascia.
Fascia transversalis: Fascial layer lining the inner surface of anterior abdominal wall, forms internal spermatic fascia when covering spermatic cord.
Colle's fascia: Superficial perineal fascia, continuous with Scarpa's fascia, not involved in spermatic cord layers.
Clinical relevance: Knowledge of fascia layers is crucial during hernia repair, hydrocele surgery, and orchidopexy to prevent injury.
Spermatic cord: Contains vas deferens, testicular artery, pampiniform plexus, lymphatics, nerves, and fasciae layers.
Lead Question - 2013
Internal spermatic fascia is derived from ?
a) External oblique muscle
b) Internal oblique muscle
c) Fascia transversalis
d) Colle's fascia
Explanation: Internal spermatic fascia is derived from the fascia transversalis as the testis descends through the deep inguinal ring. It lies deep to the cremasteric fascia and provides a protective covering around the spermatic cord. Correct answer is c) Fascia transversalis.
Guessed Question 2
Cremasteric fascia is derived from?
a) External oblique
b) Internal oblique
c) Transversalis fascia
d) Colle's fascia
Explanation: Cremasteric fascia arises from internal oblique muscle and carries cremasteric muscle fibers around spermatic cord and testis. It allows reflex elevation of testis. Correct answer is b) Internal oblique.
Guessed Question 3
External spermatic fascia is derived from?
a) External oblique aponeurosis
b) Internal oblique
c) Transversalis fascia
d) Dartos fascia
Explanation: External spermatic fascia is derived from the aponeurosis of external oblique muscle, lying superficial to cremasteric fascia, forming outer covering of spermatic cord. Correct answer is a) External oblique aponeurosis.
Guessed Question 4
Dartos fascia is continuous with?
a) Colle's fascia
b) Internal oblique
c) Transversalis fascia
d) External oblique
Explanation: Dartos fascia (superficial fascia of scrotum) is continuous with Colle's fascia in perineum and Scarpa's fascia of abdomen. It contains smooth muscle fibers. Correct answer is a) Colle's fascia.
Guessed Question 5
Layer immediately deep to external spermatic fascia?
a) Cremasteric fascia
b) Internal spermatic fascia
c) Tunica vaginalis
d) Dartos fascia
Explanation: Cremasteric fascia, containing cremasteric muscle, lies deep to external spermatic fascia and superficial to internal spermatic fascia, providing reflexive testicular elevation. Correct answer is a) Cremasteric fascia.
Guessed Question 6
Deep inguinal ring is an opening in?
a) Transversalis fascia
b) External oblique
c) Internal oblique
d) Rectus sheath
Explanation: Deep inguinal ring is an opening in the transversalis fascia, allowing passage of spermatic cord (male) or round ligament (female). Internal spermatic fascia derives from this fascia. Correct answer is a) Transversalis fascia.
Guessed Question 7
Which fascia surrounds testis directly?
a) Internal spermatic fascia
b) Cremasteric fascia
c) External spermatic fascia
d) Dartos fascia
Explanation: Internal spermatic fascia lies immediately around spermatic cord and testis, derived from transversalis fascia, providing protective layer. Correct answer is a) Internal spermatic fascia.
Guessed Question 8
Which fascia layer contains cremasteric muscle?
a) Cremasteric fascia
b) External spermatic fascia
c) Internal spermatic fascia
d) Dartos fascia
Explanation: Cremasteric fascia contains cremasteric muscle fibers originating from internal oblique, responsible for testicular elevation. Correct answer is a) Cremasteric fascia.
Guessed Question 9
Spermatic cord layers in order from superficial to deep?
a) External spermatic, cremasteric, internal spermatic
b) Internal spermatic, cremasteric, external spermatic
c) Cremasteric, external spermatic, internal spermatic
d) External spermatic, internal spermatic, cremasteric
Explanation: Layers from superficial to deep: external spermatic fascia (external oblique), cremasteric fascia (internal oblique), internal spermatic fascia (transversalis fascia). This knowledge is crucial in hernia and orchidopexy procedures. Correct answer is a) External spermatic, cremasteric, internal spermatic.
Guessed Question 10
Tunica vaginalis is derived from?
a) Processus vaginalis
b) Internal oblique
c) External oblique
d) Transversalis fascia
Explanation: Tunica vaginalis forms from the processus vaginalis, a peritoneal outpouching descending with testis. It surrounds testis and epididymis, separate from fascia layers. Correct answer is a) Processus vaginalis.
Guessed Question 11
Which fascia is clinically relevant in hydrocele surgery?
a) Internal spermatic fascia
b) Cremasteric fascia
c) External spermatic fascia
d) Dartos fascia
Explanation: Internal spermatic fascia is incised during hydrocelectomy to access tunica vaginalis. Understanding fascia layers prevents injury to cord structures. Correct answer is a) Internal spermatic fascia.
Chapter: Anatomy
Topic: Male Pelvis
Subtopic: Prostate Support & Pelvic Floor Muscles
Keyword Definitions:
Prostate: Male accessory sex gland located below the bladder, surrounding prostatic urethra, secretes seminal fluid.
Pubococcygeus: Part of levator ani muscle, supports pelvic viscera including prostate and bladder neck.
Levator ani: Group of muscles (pubococcygeus, iliococcygeus, puborectalis) forming pelvic diaphragm, supports pelvic organs.
Ischiococcygeus (coccygeus): Posterolateral pelvic floor muscle, stabilizes coccyx, assists levator ani.
Iliococcygeus: Part of levator ani, supports pelvic viscera posteriorly.
Clinical relevance: Pelvic floor muscles prevent prolapse of prostate and bladder; important in prostatectomy and urinary continence.
Perineal body: Fibromuscular structure connecting levator ani, perineal muscles, and external anal sphincter, supports prostate anteriorly.
Lead Question - 2013
Support of prostate is ?
a) Pubococcygeus
b) Ischiococcygeus
c) Ilioccygeus
d) None of the above
Explanation: The pubococcygeus muscle, part of the levator ani, forms the anterior pelvic floor and directly supports the prostate. It maintains prostate position, contributes to continence, and prevents prolapse. Correct answer is a) Pubococcygeus.
Guessed Question 2
Which muscle elevates pelvic floor and supports bladder?
a) Pubococcygeus
b) Coccygeus
c) Iliococcygeus
d) External anal sphincter
Explanation: Pubococcygeus elevates the pelvic floor and supports bladder and prostate. Strengthening it improves urinary continence. Correct answer is a) Pubococcygeus.
Guessed Question 3
Posterior pelvic floor muscle stabilizing coccyx is?
a) Pubococcygeus
b) Coccygeus
c) Iliococcygeus
d) Rectococcygeus
Explanation: Coccygeus (ischiococcygeus) lies posterolaterally, stabilizing the coccyx and assisting levator ani in pelvic support. Correct answer is b) Coccygeus.
Guessed Question 4
Which part of levator ani is more posterior?
a) Pubococcygeus
b) Iliococcygeus
c) Puborectalis
d) Coccygeus
Explanation: Iliococcygeus forms posterior part of levator ani, supporting rectum and pelvic viscera posteriorly. Correct answer is b) Iliococcygeus.
Guessed Question 5
Pelvic diaphragm is formed by?
a) Levator ani + Coccygeus
b) Rectus abdominis
c) Obturator internus
d) Gluteus maximus
Explanation: Pelvic diaphragm consists of levator ani (pubococcygeus, iliococcygeus, puborectalis) and coccygeus muscles, providing support to pelvic organs. Correct answer is a) Levator ani + Coccygeus.
Guessed Question 6
Perineal body connects all except?
a) Levator ani
b) Bulbospongiosus
c) Puborectalis
d) Coccygeus
Explanation: Perineal body connects levator ani, external anal sphincter, and perineal muscles, but not coccygeus directly. Supports prostate and pelvic floor. Correct answer is d) Coccygeus.
Guessed Question 7
Which muscle contributes to urinary continence?
a) Pubococcygeus
b) Coccygeus
c) Iliococcygeus
d) Gluteus maximus
Explanation: Pubococcygeus supports urethra and prostate, maintains urethral closure, aiding urinary continence. Correct answer is a) Pubococcygeus.
Guessed Question 8
Posterior boundary of pelvic floor is formed by?
a) Coccygeus
b) Pubococcygeus
c) Iliococcygeus
d) Bulbospongiosus
Explanation: Coccygeus (ischiococcygeus) forms posterior boundary of pelvic floor, supporting sacrum and coccyx. Correct answer is a) Coccygeus.
Guessed Question 9
Which muscle is most anterior in levator ani?
a) Pubococcygeus
b) Iliococcygeus
c) Coccygeus
d) Puborectalis
Explanation: Pubococcygeus lies anteriorly, forming main support for prostate, bladder, and urethra. Correct answer is a) Pubococcygeus.
Guessed Question 10
Which muscle attaches to pubic bone and coccyx?
a) Pubococcygeus
b) Iliococcygeus
c) Coccygeus
d) External anal sphincter
Explanation: Pubococcygeus originates from pubic bone and inserts into coccyx and anococcygeal raphe, supporting prostate and pelvic viscera. Correct answer is a) Pubococcygeus.
Guessed Question 11
Which muscle forms central part of pelvic floor?
a) Pubococcygeus
b) Iliococcygeus
c) Coccygeus
d) Obturator internus
Explanation: Pubococcygeus forms central part of pelvic floor, supporting prostate, bladder, and urethra, crucial in male pelvic anatomy. Correct answer is a) Pubococcygeus.
Chapter: Anatomy
Topic: Male Reproductive System
Subtopic: Testicular Anatomy & Position
Keyword Definitions:
Testis: Male gonad responsible for spermatogenesis and androgen secretion.
Spermatic cord: Contains vas deferens, testicular artery, pampiniform plexus, lymphatics, and nerves.
Cremaster muscle: Elevates testis for temperature regulation.
Pampiniform plexus: Venous network surrounding testicular artery, helps thermoregulation.
Scrotum: Skin pouch housing testes, maintaining lower temperature than body for spermatogenesis.
Clinical relevance: Higher position of right testis is normal; important in cryptorchidism evaluation and orchidopexy planning.
Cryptorchidism: Undescended testis, more common on right side; may cause infertility and malignancy risk.
Lead Question - 2013
Location of testis is higher on ?
a) Right side
b) Left side
c) May be on right or left side
d) Same level on both sides
Explanation: Anatomically, the right testis usually hangs slightly higher than the left within the scrotum due to variations in spermatic cord length. This is normal and clinically important to distinguish from pathological causes like hydrocele or cryptorchidism. Correct answer is a) Right side.
Guessed Question 2
Which testis is more commonly undescended in cryptorchidism?
a) Right
b) Left
c) Both equally
d) None
Explanation: Cryptorchidism is more frequently observed on the right side, possibly due to delayed descent or shorter gubernacular attachment. Early detection is essential to prevent infertility and malignancy. Correct answer is a) Right.
Guessed Question 3
The testis descends during which gestational period?
a) 7–8 weeks
b) 12–14 weeks
c) 28–32 weeks
d) After birth
Explanation: Testicular descent occurs in two phases: transabdominal (7–12 weeks) and inguinoscrotal (28–32 weeks). This ensures the testes reach scrotum for optimal spermatogenesis. Correct answer is c) 28–32 weeks.
Guessed Question 4
Which structure guides testicular descent?
a) Gubernaculum
b) Cremaster muscle
c) Vas deferens
d) Epididymis
Explanation: The gubernaculum anchors the testis and guides its passage from the abdomen to scrotum during fetal development. Failure of this leads to cryptorchidism. Correct answer is a) Gubernaculum.
Guessed Question 5
Normal testis temperature is maintained at?
a) Same as body
b) 2–4°C lower than body
c) 2–4°C higher
d) Variable
Explanation: Scrotum maintains testicular temperature 2–4°C below body temperature, essential for spermatogenesis. Cremaster and pampiniform plexus regulate this. Correct answer is b) 2–4°C lower than body.
Guessed Question 6
Which side is more prone to varicocele?
a) Left
b) Right
c) Both equally
d) None
Explanation: Varicocele occurs more commonly on the left due to longer left testicular vein draining into left renal vein at a right angle, increasing venous pressure. Correct answer is a) Left.
Guessed Question 7
Right testis is higher due to?
a) Shorter spermatic cord
b) Longer spermatic cord
c) Smaller size
d) Heavier weight
Explanation: Right testis is slightly higher because the right spermatic cord is usually shorter than the left, resulting in a higher scrotal position. Correct answer is a) Shorter spermatic cord.
Guessed Question 8
Which muscle elevates the testis?
a) Cremaster
b) Dartos
c) External oblique
d) Rectus abdominis
Explanation: Cremaster muscle contracts reflexively to elevate the testis toward the body for thermoregulation and protection. Important in cremasteric reflex testing. Correct answer is a) Cremaster.
Guessed Question 9
Scrotal septum separates:
a) Right & left testis
b) Testis & epididymis
c) Testis & spermatic cord
d) Epididymis & vas deferens
Explanation: Scrotal septum divides scrotum into right and left compartments, each containing a testis and epididymis, preventing torsion spread. Correct answer is a) Right & left testis.
Guessed Question 10
Pampiniform plexus function is?
a) Thermoregulation
b) Hormone secretion
c) Sperm transport
d) Scrotal support
Explanation: Pampiniform plexus of veins surrounds testicular artery to cool arterial blood, maintaining ideal temperature for spermatogenesis. Correct answer is a) Thermoregulation.
Guessed Question 11
Testicular position abnormality is called?
a) Cryptorchidism
b) Varicocele
c) Hydrocele
d) Orchitis
Explanation: Cryptorchidism refers to undescended testis, more often on the right. It increases infertility and malignancy risk, requiring early intervention. Correct answer is a) Cryptorchidism.
Chapter: Anatomy
Topic: Pelvis
Subtopic: Lymphatic Drainage of Female Reproductive Organs
Keyword Definitions:
Para-aortic lymph nodes: Lymph nodes along the abdominal aorta, draining ovaries, uterine tubes, and upper uterus.
External iliac lymph nodes: Nodes along external iliac vessels, draining upper bladder, cervix, and upper vagina.
Superior inguinal lymph nodes: Located in femoral triangle, draining lower vulva, lower vagina, and superficial structures of lower limb.
Deep inguinal lymph nodes: Beneath fascia lata, draining glans penis/clitoris, deep lower limb structures.
Pelvic lymph nodes: Network including obturator, internal iliac, external iliac, and sacral nodes.
Clinical relevance: Knowledge of lymphatic drainage is vital for staging cervical and vaginal cancers and planning surgery or radiotherapy.
Lead Question - 2013
Which lymph nodes drain upper vagina & cervix?
a) Para aortic
b) External iliac
c) Superior inguinal
d) Deep inguinal
Explanation: The upper vagina and cervix primarily drain into the external iliac lymph nodes, with some drainage to internal iliac and obturator nodes. Para-aortic nodes mainly drain ovaries and uterine tubes. Correct answer is b) External iliac.
Guessed Question 2
Lower vagina primarily drains into?
a) External iliac nodes
b) Internal iliac nodes
c) Superior inguinal nodes
d) Para-aortic nodes
Explanation: The lower vagina and vulva drain primarily into superficial and superior inguinal lymph nodes, providing a pathway for potential metastasis. Correct answer is c) Superior inguinal nodes.
Guessed Question 3
Ovaries drain mainly to which lymph nodes?
a) External iliac
b) Internal iliac
c) Para-aortic
d) Inguinal
Explanation: Ovarian lymphatics follow the ovarian vessels to the para-aortic (lumbar) lymph nodes near the renal vessels. Correct answer is c) Para-aortic.
Guessed Question 4
Cervical cancer commonly metastasizes to which nodes first?
a) Para-aortic
b) External iliac
c) Superior inguinal
d) Sacral
Explanation: Early cervical cancer spreads to the external iliac, internal iliac, and obturator nodes. Para-aortic and sacral involvement occurs later. Correct answer is b) External iliac.
Guessed Question 5
Which lymph nodes lie along obturator vessels?
a) Obturator nodes
b) External iliac nodes
c) Superior inguinal nodes
d) Para-aortic nodes
Explanation: Obturator lymph nodes are situated along the obturator vessels, draining the pelvic floor, bladder, cervix, and upper vagina. Correct answer is a) Obturator nodes.
Guessed Question 6
Deep inguinal lymph nodes receive drainage from?
a) Lower limb
b) Upper vagina
c) Cervix
d) Para-aortic nodes
Explanation: Deep inguinal nodes lie beneath fascia lata and receive lymph from the lower limb, glans penis/clitoris, and deep structures of the lower pelvis. Correct answer is a) Lower limb.
Guessed Question 7
Which nodes are involved in vulvar carcinoma?
a) Para-aortic
b) Superior inguinal
c) External iliac
d) Obturator
Explanation: Vulvar carcinoma primarily drains to superficial and superior inguinal lymph nodes, which are first sites of metastasis. Correct answer is b) Superior inguinal.
Guessed Question 8
Internal iliac lymph nodes drain?
a) Upper bladder, cervix, uterus
b) Lower limb
c) Ovaries
d) Lower vagina only
Explanation: Internal iliac nodes drain the cervix, upper vagina, bladder, and uterus, forming an important part of pelvic lymphatic network. Correct answer is a) Upper bladder, cervix, uterus.
Guessed Question 9
Para-aortic nodes are clinically important in?
a) Staging ovarian cancer
b) Breast cancer
c) Colon cancer
d) Thyroid cancer
Explanation: Para-aortic lymph nodes receive lymph from ovaries, uterine tubes, and upper uterus, serving as a key site in staging ovarian and some uterine cancers. Correct answer is a) Staging ovarian cancer.
Guessed Question 10
Which pelvic node group is commonly biopsied in cervical cancer?
a) External iliac
b) Para-aortic
c) Superior inguinal
d) Deep inguinal
Explanation: External iliac nodes are commonly biopsied or sampled during pelvic lymphadenectomy in cervical cancer due to their early involvement. Correct answer is a) External iliac.
Guessed Question 11
Obturator nodes are located in relation to?
a) Obturator vessels
b) External iliac vessels
c) Internal thoracic artery
d) Para-aortic region
Explanation: Obturator lymph nodes lie along obturator vessels in the obturator fossa and are part of the primary drainage pathway of the cervix and upper vagina. Correct answer is a) Obturator vessels.
Chapter: Obstetrics & Gynecology
Topic: Female Pelvic Anatomy & Oncology
Subtopic: Lymphatic Drainage of Cervix & Clinical Implications
Keywords (Definitions)
Cervix: Lower part of uterus opening into the vagina; key site for HPV-related malignancy.
Lymphatic drainage: Network conveying lymph from tissues to regional lymph nodes.
Iliac lymph nodes: Pelvic nodes along external, internal, and common iliac vessels receiving cervical lymph.
External iliac nodes: Nodes along external iliac vessels; frequent first-echelon nodes from cervix.
Internal iliac (hypogastric) nodes: Pelvic nodes draining cervix via paracervical pathways.
Obturator nodes: Nodes in obturator fossa around obturator nerve; common sentinel basin.
Sacral nodes: Lateral/ presacral nodes receiving posterior cervical lymph.
Para-aortic (lumbar) nodes: Nodes along aorta; second-echelon or advanced spread from pelvis.
Inguinal nodes (superficial/deep): Groin nodes; drain vulva and lower third of vagina, not primary cervix.
Sentinel lymph node (SLN): First draining node(s) from a tumor; used for targeted sampling/mapping.
FIGO 2018 IIIC stage: Cervical cancer staging: IIIC1 pelvic node metastasis; IIIC2 para-aortic.
PET-CT: Imaging modality sensitive for nodal metastasis, especially para-aortic.
Upper vs lower vagina drainage: Upper to pelvic nodes; lower to inguinal nodes.
Radical hysterectomy (Type C1): Nerve-sparing resection with parametrial and pelvic lymphadenectomy.
Lymphocyst: Post-lymphadenectomy lymph collection in pelvis/retroperitoneum.
Obturator nerve: Landmark within obturator fossa; guides identification of obturator nodes.
Lead Question - 2012
Lymphatic drainage of cervix is to
a) Iliac lymph nodes
b) Para aortic lymph nodes
c) Superficial inguinal lymph nodes
d) Deep inguinal lymph nodes
Explanation (≈50 words): The primary lymphatic drainage of the cervix is to the pelvic (iliac) nodal groups—obturator, internal iliac, external iliac, and sacral. Inguinal nodes drain the lower third of the vagina and vulva; para-aortic nodes are second-echelon spread. Answer: a) Iliac lymph nodes.
Guessed MCQ 1
A 36-year-old with FIGO IA2 cervical cancer undergoes SLN mapping. Which tracer has the best bilateral detection in experienced hands?
a) Indigo carmine
b) Indocyanine green (ICG)
c) Trypan blue
d) Methylene blue
Explanation (≈50 words): Indocyanine green with near-infrared imaging achieves high bilateral sentinel detection and low false-negative rates in early cervical cancer. Blue dyes alone have lower sensitivity. Technetium may be combined but ICG is widely preferred for real-time visualization. Answer: b) Indocyanine green (ICG).
Guessed MCQ 2
First-echelon nodal basin most commonly involved in carcinoma cervix is
a) Popliteal nodes
b) Axillary nodes
c) Obturator nodes
d) Deep inguinal nodes
Explanation: Lymph from the cervix passes through paracervical channels to obturator and internal/external iliac nodes. Obturator nodes in the obturator fossa are the commonest first-echelon group sampled during pelvic lymphadenectomy or SLN biopsy. Popliteal and axillary nodes are unrelated; deep inguinal nodes drain lower limb and perineum. Answer: c) Obturator nodes.
Guessed MCQ 3 (Clinical)
A 48-year-old with bulky cervical mass has PET-CT showing FDG-avid common iliac nodes but no para-aortic uptake. FIGO 2018 stage is
a) IIB
b) IIIC1
c) IIIC2
d) IVA
Explanation: Nodal staging in FIGO 2018 classifies pelvic nodal metastasis (including common iliac) as stage IIIC1, while para-aortic nodal involvement is IIIC2. Local parametrial involvement defines IIB, and invasion of adjacent organs bladder/rectum indicates IVA. Here only pelvic nodes are positive. Answer: b) IIIC1.
Guessed MCQ 4 (Clinical)
A patient with cervical cancer and lower third vaginal involvement is likely to have additional drainage to
a) Superficial inguinal nodes
b) Mediastinal nodes
c) Popliteal nodes
d) Epitrochlear nodes
Explanation: The lower third of the vagina drains to superficial inguinal nodes, creating a pathway for groin metastasis when the disease extends inferiorly. Mediastinal, popliteal, and epitrochlear nodes are not involved in genital tract drainage. Hence groin evaluation is important if the lower vagina is affected. Answer: a) Superficial inguinal nodes.
Guessed MCQ 5 (Clinical)
Post-radical hysterectomy, histology shows metastasis in para-aortic nodes only. FIGO 2018 stage is
a) IIIC1
b) IIIC2
c) IIIA
d) IVB
Explanation: Isolated para-aortic nodal metastasis without distant organ spread upgrades to FIGO IIIC2. IIIC1 denotes pelvic nodal disease. IIIA involves lower vaginal invasion; IVB implies distant metastases beyond the abdomen/pelvis (e.g., lung, bone). Para-aortic positivity alone fits IIIC2. Answer: b) IIIC2.
Guessed MCQ 6
Best single imaging modality to detect occult para-aortic nodal metastasis pre-treatment in cervical cancer
a) Pelvic ultrasound
b) PET-CT
c) Plain CT
d) Chest X-ray
Explanation (≈50 words): PET-CT outperforms CT and MRI for detecting metabolically active nodal metastases, particularly in para-aortic chains, guiding field extension for chemoradiation. Ultrasound and chest X-ray lack sensitivity for retroperitoneal nodal disease. Tissue confirmation may still be required when management will change. Answer: b) PET-CT.
Guessed MCQ 7
Primary lymphatic drainage of the upper vagina is mainly to
a) External/internal iliac nodes
b) Superficial inguinal nodes
c) Axillary nodes
d) Popliteal nodes
Explanation: The upper two-thirds of the vagina drain predominantly to the internal and external iliac nodes, paralleling cervical drainage. The lower third drains to the superficial inguinal nodes. Axillary and popliteal nodes are unrelated to pelvic genital tract lymphatics. Answer: a) External/internal iliac nodes.
Guessed MCQ 8
Which surgical procedure routinely addresses parametrial tissue and pelvic nodes in operable cervical cancer?
a) Simple hysterectomy
b) Radical hysterectomy (Type C1, nerve-sparing)
c) Myomectomy
d) Endometrial ablation
Explanation: Radical hysterectomy Type C1 (Querleu–Morrow) removes uterus with parametria and includes pelvic lymphadenectomy while preserving pelvic nerves. Simple hysterectomy lacks adequate margins and nodal assessment; myomectomy and ablation are not oncologic procedures. Answer: b) Radical hysterectomy (Type C1, nerve-sparing).
Guessed MCQ 9
Posterior cervical lymph primarily drains to which nodal group?
a) Presacral/lateral sacral nodes
b) Axillary nodes
c) Epitrochlear nodes
d) Deep inguinal nodes
Explanation: The posterior cervix drains via uterosacral pathways to presacral and lateral sacral nodes, part of the pelvic (iliac–sacral) chains. Axillary and epitrochlear nodes are upper-limb related; deep inguinal nodes pertain to lower limb and perineum. Answer: a) Presacral/lateral sacral nodes.
Guessed MCQ 10 (Clinical)
After pelvic lymphadenectomy for early cervical cancer, a patient develops a painless pelvic mass causing leg edema. Most likely complication is
a) Hematoma
b) Lymphocyst
c) Abscess
d) Seroma from abdominal wall
Explanation: Disruption of pelvic lymphatics can lead to lymphocyst formation—an encapsulated lymph collection in the retroperitoneum causing mass effect and lower-limb edema or hydronephrosis. Hematoma or abscess are typically painful and inflammatory; abdominal wall seroma is superficial. Answer: b) Lymphocyst.
Guessed MCQ 11
During node dissection, which anatomic landmark confirms entry into the obturator fossa containing the obturator nodal packet?
a) Femoral artery
b) Obturator nerve
c) Round ligament
d) Ureteric orifice
Explanation: The obturator nerve traverses the obturator fossa and serves as a key landmark for identifying and clearing obturator nodes during pelvic lymphadenectomy. Femoral artery is outside the pelvis; round ligament is anterior; ureteric orifice relates to bladder trigone, not the obturator space. Answer: b) Obturator nerve..
Chapter: Female Pelvis & Perineum | Topic: External Genitalia | Subtopic: Bartholin (Greater Vestibular) Gland
Keyword Definitions
Bartholin (greater vestibular) gland: Mucus-secreting gland posterolateral to vaginal orifice, opens into vestibule at 4 and 8 o’clock.
Superficial perineal pouch: Space between Colles fascia and perineal membrane; contains Bartholin glands in females.
Deep perineal pouch: Space superior to perineal membrane; contains external urethral sphincter complex.
Perineal membrane: Fibrous sheet forming floor of deep pouch and roof of superficial pouch.
Ischioanal (ischiorectal) fossa: Fat-filled wedge lateral to anal canal; not the location of Bartholin glands.
Vestibule of vagina: Area between labia minora containing urethral and vaginal openings and Bartholin ducts.
Skene’s (paraurethral) glands: Mucus glands near urethral meatus; distinct from Bartholin glands.
Word catheter: Temporary drain for Bartholin cyst/abscess after incision to maintain duct patency.
Marsupialization: Procedure suturing cyst wall to vestibular mucosa to create permanent opening.
Inguinal lymph nodes: Primary drainage for vulva and Bartholin region.
Lead Question – 2012
Bartholin gland situated in ?
a) Superficial perineal pouch
b) Deep perineal pouch
c) Inguinal canal
d) Ischiorecal fossa
Explanation (Answer: a)
Bartholin glands lie in the superficial perineal pouch, between Colles fascia and the perineal membrane, posterolateral to the vaginal introitus. Their ducts open into the vestibule at 4 and 8 o’clock. They are not in the deep perineal pouch, inguinal canal, or ischioanal fossa.
1) A 26-year-old presents with a tender swelling at 7 o’clock of the vestibule. Which anatomical space contains the affected gland?
a) Superficial perineal pouch
b) Deep perineal pouch
c) Obturator canal
d) Paravaginal space
Explanation (Answer: a)
Bartholin abscess classically arises from glands in the superficial perineal pouch, inferior to the perineal membrane. Deep pouch houses urethral sphincter complex; obturator canal transmits neurovascular structures; paravaginal space relates to pelvic fascia, not vestibular glands.
2) During incision and drainage of a Bartholin abscess, the surgeon aims the incision toward the duct opening. Typical duct opening position?
a) 12 o’clock of urethral meatus
b) 4 and 8 o’clock positions in vestibule
c) Lateral to clitoral frenulum
d) Within the hymenal ring at 2 o’clock
Explanation (Answer: b)
Bartholin ducts open into the vestibule at approximately 4 and 8 o’clock near the posterior introitus. They are not adjacent to the urethral meatus or clitoris. Recognizing precise openings guides drainage and placement of a Word catheter to prevent recurrence.
3) Lymphatic spread from a carcinoma arising in the Bartholin gland primarily involves which nodal basin first?
a) External iliac nodes
b) Inguinal nodes
c) Para-aortic nodes
d) Obturator nodes
Explanation (Answer: b)
The vulvar region, including Bartholin glands, drains chiefly to superficial and deep inguinal lymph nodes. Pelvic nodes such as external iliac or obturator may be involved secondarily. Accurate mapping influences staging, imaging, and surgical management in suspected Bartholin gland malignancy.
4) A painless, fluctuant 3-cm vestibular cyst near 5 o’clock recurs after simple aspiration. Best next step?
a) Marsupialization
b) Empirical pelvic lymphadenectomy
c) Excision of entire gland in clinic
d) Broad-spectrum antibiotics alone
Explanation (Answer: a)
Recurrent Bartholin duct cysts are managed by marsupialization or Word catheter placement to create a permanent drainage tract. Routine lymphadenectomy is inappropriate. Office gland excision is not first line. Antibiotics alone won’t address duct obstruction causing recurrence.
5) A 33-year-old with cellulitis around a Bartholin abscess asks the nerve supply of the painful area. Principal somatic nerve?
a) Pudendal nerve
b) Iliohypogastric nerve
c) Genitofemoral nerve (genital branch)
d) Obturator nerve
Explanation (Answer: a)
Somatic innervation of the perineum and vestibule is mainly via the pudendal nerve and its branches. Iliohypogastric and genitofemoral supply anterior abdominal wall and labia majora skin partly, while obturator serves medial thigh, not the vestibular mucosa.
6) Which artery most directly supplies the Bartholin gland region?
a) External pudendal artery
b) Internal pudendal artery branches
c) Uterine artery
d) Inferior epigastric artery
Explanation (Answer: b)
The perineum and vestibular structures, including Bartholin glands, receive blood mainly from branches of the internal pudendal artery. External pudendal supplies superficial vulvar skin. Uterine and inferior epigastric arteries do not primarily perfuse the vestibular gland region.
7) A 45-year-old with a new Bartholin mass should be evaluated for malignancy. Which statement supports biopsy consideration?
a) Any new Bartholin mass after age 40 merits evaluation
b) Malignancy never arises in this gland
c) Age is irrelevant; ignore unless febrile
d) Only bilateral masses are concerning
Explanation (Answer: a)
New Bartholin gland masses in women over 40 warrant biopsy or excision to exclude adenocarcinoma or squamous carcinoma. Although rare, cancer occurs. Age and new onset guide suspicion; fever and bilaterality do not rule malignancy in or out.
8) A vestibular swelling discharges through a small mucosal opening after I&D. Which device best maintains duct patency during healing?
a) Foley catheter
b) Word catheter
c) Penrose drain
d) T-tube
Explanation (Answer: b)
A Word catheter is specifically designed for Bartholin duct cyst/abscess, with a small balloon that keeps the new tract patent for weeks, allowing epithelialization. Foley and Penrose are less suitable; T-tubes are for biliary/airway applications, not vestibular ducts.
9) Which structure forms the superior boundary (roof) of the superficial perineal pouch containing Bartholin glands?
a) Colles fascia
b) Perineal membrane
c) Superficial fascia of abdomen
d) Levator ani
Explanation (Answer: b)
The superficial perineal pouch lies between the perineal membrane (roof) and Colles fascia (floor). Levator ani is superior to the deep pouch. Recognizing these boundaries is essential for safe incision placement during drainage of Bartholin pathology.
10) A tender vestibular swelling is mistaken for a Skene’s gland infection. Which finding favors Bartholin origin?
a) Discharge from urethral meatus
b) Swelling at 4 or 8 o’clock near posterior introitus
c) Pain localized above clitoris
d) Mass along lateral vaginal fornix
Explanation (Answer: b)
Skene’s glands open near the urethral meatus anteriorly. Bartholin swellings localize posterolaterally at the introitus, typically 4 or 8 o’clock, within the superficial perineal pouch. Fornix masses suggest Gartner duct or paravaginal cysts, not Bartholin pathology.
11) After wide local excision of a Bartholin tumor, which early nodal assessment is most anatomically justified?
a) Sentinel mapping to para-aortic nodes
b) Inguinal node assessment first
c) Primary obturator node dissection
d) Exclusively presacral node sampling
Explanation (Answer: b)
Vulvar and vestibular lymphatics, including Bartholin glands, drain initially to the inguinal nodes. Therefore, early nodal assessment focuses on superficial/deep inguinal basins. Para-aortic, obturator, and presacral nodes are secondary considerations guided by stage, imaging, and pathologic risk factors.
Chapter: Abdomen & Pelvis
Topic: Female Reproductive System
Subtopic: Ovarian Fossa
Keyword Definitions:
Ovarian fossa – Depression on lateral pelvic wall where the ovary rests.
Internal iliac artery – Major pelvic artery forming the posterior boundary of ovarian fossa.
Ureter – Muscular tube carrying urine, forming part of the floor of the fossa.
Obliterated umbilical artery – Remnant of fetal circulation, forms anterior boundary of fossa.
Round ligament of ovary – Incorrect term; true ligament is ovarian ligament.
Lead Question – 2012
Ovarian fossa is formed by all except?
a) Obliterated umbilical artery
b) Internal iliac artery
c) Ureter
d) Round ligament of ovary
Explanation: The ovarian fossa is bounded anteriorly by the obliterated umbilical artery, posteriorly by the internal iliac artery and ureter. The round ligament of ovary does not exist (confusion with ovarian ligament). Hence, the correct answer is d) Round ligament of ovary.
Guessed Questions for NEET PG
1. Ovary is supplied mainly by?
a) Uterine artery
b) Ovarian artery
c) Vaginal artery
d) Inferior epigastric artery
Explanation: The ovary receives its main blood supply from the ovarian artery, a direct branch of the abdominal aorta. The uterine artery provides anastomotic supply. Correct answer is b) Ovarian artery.
2. Venous drainage of ovary is?
a) Directly into IVC (right), renal vein (left)
b) Both into renal veins
c) Both into IVC
d) Into iliac veins
Explanation: The right ovarian vein drains directly into the IVC, while the left drains into the left renal vein. This asymmetry has clinical significance in varicocele. Correct answer is a).
3. Lymphatic drainage of ovary is?
a) Superficial inguinal nodes
b) External iliac nodes
c) Para-aortic nodes
d) Internal iliac nodes
Explanation: The ovary develops in the lumbar region and descends into pelvis. Its lymph drains into para-aortic (lumbar) nodes, important for staging ovarian cancer. Correct answer is c).
4. Which nerve is closely related to ovarian fossa?
a) Genitofemoral
b) Obturator
c) Femoral
d) Pudendal
Explanation: The obturator nerve runs along the lateral pelvic wall beneath the ovarian fossa, making it vulnerable during pelvic surgery. Correct answer is b).
5. A patient with ovarian carcinoma presents with enlarged para-aortic nodes. This is because of?
a) Direct spread
b) Lymphatic drainage
c) Venous spread
d) Peritoneal spread
Explanation: The ovary’s lymphatics drain to para-aortic nodes, explaining enlargement in malignancy. Correct answer is b) Lymphatic drainage.
6. Which structure does NOT pass through the broad ligament?
a) Round ligament of uterus
b) Ovarian ligament
c) Ureter
d) Ovarian vessels
Explanation: The ureter runs under the broad ligament but does not pass through it. Other structures are enclosed within folds of the broad ligament. Correct answer is c) Ureter.
7. During oophorectomy, which structure is most at risk of injury at infundibulopelvic ligament?
a) Ureter
b) Internal iliac artery
c) External iliac vein
d) Femoral nerve
Explanation: The ureter lies close to the infundibulopelvic ligament (suspensory ligament of ovary). Surgical clamping risks ureteral injury. Correct answer is a) Ureter.
8. Pain of ovarian torsion is referred to?
a) Umbilical region
b) Suprapubic region
c) Shoulder tip
d) Left hypochondrium
Explanation: Ovarian pain is referred to the umbilical region via T10 spinal segments, same as appendix. Correct answer is a).
9. Which ligament contains ovarian vessels?
a) Broad ligament
b) Ovarian ligament
c) Infundibulopelvic ligament
d) Round ligament
Explanation: The infundibulopelvic ligament (suspensory ligament of ovary) carries ovarian vessels from aorta to ovary. Correct answer is c).
10. Ovary develops from which embryological structure?
a) Mesonephric duct
b) Paramesonephric duct
c) Genital ridge
d) Cloaca
Explanation: The ovary develops from the genital ridge, formed by coelomic epithelium and underlying mesenchyme. Correct answer is c).
Topic: Uterine Support
Subtopic: Ligamentous Support of Uterus
Keyword Definitions:
Cardinal Ligament: Primary ligament providing lateral support to the uterus, extends from cervix to lateral pelvic wall.
Broad Ligament: Double fold of peritoneum attaching uterus to lateral pelvic walls, contains vessels and nerves.
Round Ligament: Connects uterine horns to labia majora via inguinal canal, maintains anteverted position.
Pubocervical Ligament: Connects cervix to pubic symphysis, contributes to anterior support.
Uterosacral Ligament: Extends from cervix to sacrum, provides posterior support.
Pelvic Floor Muscles: Muscular layer supporting pelvic organs, including levator ani.
Prolapse: Descent of uterus or vaginal walls due to ligament or muscle weakness.
Parametrium: Connective tissue surrounding cervix, includes cardinal ligaments.
Anteverted Uterus: Normal position of uterus inclined forward over bladder.
Pelvic Organ Support: Combination of ligaments and muscles maintaining organ position.
Lead Question – 2012
Main support of uterus is from – ligament :
a) Cardinal
b) Broad
c) Round
d) Pubocervical
Explanation: The cardinal ligament provides primary lateral support to the uterus by anchoring the cervix and upper vagina to the lateral pelvic wall. It contains uterine vessels and connective tissue. Weakness or injury can lead to uterine prolapse. Therefore, the correct answer is a) Cardinal. Other ligaments contribute but are secondary.
1. Which ligament helps maintain the anteverted position of the uterus?
a) Cardinal
b) Broad
c) Round
d) Uterosacral
Explanation: The round ligament extends from uterine horns to labia majora via the inguinal canal, maintaining anteverted position. Weakness can allow retroversion. Correct answer: c) Round.
2. Uterosacral ligaments provide which type of uterine support?
a) Anterior
b) Lateral
c) Posterior
d) Inferior
Explanation: The uterosacral ligaments extend from cervix to sacrum and provide posterior support, preventing backward displacement. They are clinically important in uterine prolapse surgeries. Correct answer: c) Posterior.
3. Broad ligament contains which of the following structures?
a) Uterine vessels
b) Ovarian vessels
c) Nerves and lymphatics
d) All of the above
Explanation: The broad ligament is a double layer of peritoneum attaching uterus to lateral pelvic walls. It contains uterine and ovarian vessels, nerves, and lymphatics. Correct answer: d) All of the above.
4. A 50-year-old woman presents with uterovaginal prolapse. Weakness of which ligament is most likely responsible?
a) Round ligament
b) Cardinal ligament
c) Broad ligament
d) Pubocervical ligament
Explanation: Uterovaginal prolapse is most commonly caused by weakness of cardinal ligaments. These ligaments provide primary lateral support. Damage occurs due to childbirth trauma or aging. Correct answer: b) Cardinal ligament.
5. Which ligament connects cervix to pubic symphysis?
a) Pubocervical ligament
b) Cardinal ligament
c) Round ligament
d) Uterosacral ligament
Explanation: The pubocervical ligament connects cervix and upper vagina to pubic symphysis, providing anterior support. Weakening contributes to anterior vaginal wall prolapse (cystocele). Correct answer: a) Pubocervical ligament.
6. During hysterectomy, which ligament must be carefully ligated to control uterine vessels?
a) Broad ligament
b) Cardinal ligament
c) Round ligament
d) Uterosacral ligament
Explanation: The cardinal ligament contains uterine vessels, which must be ligated during hysterectomy to prevent hemorrhage. Correct answer: b) Cardinal ligament.
7. Which ligament is most likely to be stretched during pregnancy to maintain uterine position?
a) Broad ligament
b) Round ligament
c) Cardinal ligament
d) Uterosacral ligament
Explanation: During pregnancy, the round ligament stretches as the uterus enlarges to maintain anteverted position. Stretching can cause ligamentous pain in lower abdomen. Correct answer: b) Round ligament.
8. Which structure is part of the parametrium?
a) Cardinal ligament
b) Broad ligament
c) Uterosacral ligament
d) Round ligament
Explanation: The parametrium is connective tissue surrounding cervix. The cardinal ligament is a major component, providing lateral support and containing uterine vessels. Correct answer: a) Cardinal ligament.
9. Damage to which ligament may result in retroversion of the uterus postpartum?
a) Round ligament
b) Cardinal ligament
c) Uterosacral ligament
d) Pubocervical ligament
Explanation: The round ligament maintains anteversion. Weakening postpartum or after surgery may lead to retroverted uterus, often asymptomatic. Cardinal ligament damage leads to prolapse, not retroversion. Correct answer: a) Round ligament.
10. Which ligament is primarily responsible for posterior support preventing uterine descent?
a) Cardinal ligament
b) Broad ligament
c) Uterosacral ligament
d) Round ligament
Explanation: The uterosacral ligament extends from cervix to sacrum and prevents posterior displacement and uterine descent. It is especially important in posterior vaginal wall prolapse. Correct answer: c) Uterosacral ligament.
Subtopic: Spermatogenesis
Keywords & Definitions:
Haploid: A cell with a single set of unpaired chromosomes (n).
Diploid: A cell containing two complete sets of chromosomes (2n).
Spermatogonia: Diploid stem cells in the seminiferous tubules that divide by mitosis.
Primary spermatocyte: Diploid cell formed from spermatogonia entering meiosis I.
Secondary spermatocyte: Haploid cell formed after completion of meiosis I.
Spermatid: Immature haploid sperm cell formed after meiosis II.
Spermatozoa: Mature male gametes capable of fertilizing an ovum.
Meiosis: Cell division that reduces chromosome number by half, producing haploid gametes.
Chromosome number in humans: 46 (diploid), 23 (haploid).
Gametes: Haploid reproductive cells (sperm and ova).
Q1 (2012). Haploid number of chromosomes is seen in?
a) Spermatogonia
b) Primary spermatocytes
c) Secondary spermatocyte
d) None
Explanation & Answer:
Correct answer: c) Secondary spermatocyte.
Secondary spermatocytes are haploid because they result from the first meiotic division of primary spermatocytes, reducing chromosome number from 46 (2n) to 23 (n). These cells will undergo meiosis II to form spermatids, which then differentiate into spermatozoa. Spermatogonia and primary spermatocytes are diploid.
Q2. Which stage of spermatogenesis is immediately after meiosis II?
a) Spermatogonia
b) Spermatids
c) Secondary spermatocytes
d) Spermatozoa
Explanation & Answer:
Correct answer: b) Spermatids.
Meiosis II of secondary spermatocytes produces haploid spermatids. These cells are round, non-motile, and immature. Spermiogenesis, a transformation process, converts spermatids into motile spermatozoa capable of fertilization.
Q3. In humans, how many chromosomes are present in a spermatid?
a) 46
b) 23
c) 22
d) 44
Explanation & Answer:
Correct answer: b) 23.
Spermatids are haploid cells with 23 chromosomes — 22 autosomes and either an X or Y sex chromosome. This ensures that after fertilization with the ovum (also 23 chromosomes), the zygote restores the diploid count of 46 chromosomes.
Q4 (Clinical). A male patient has a mutation that halts meiosis I. Which cell type will be absent in his testes?
a) Spermatogonia
b) Primary spermatocytes
c) Secondary spermatocytes
d) Sertoli cells
Explanation & Answer:
Correct answer: c) Secondary spermatocytes.
If meiosis I is blocked, primary spermatocytes cannot produce secondary spermatocytes. This results in infertility since later stages of sperm maturation (spermatids and spermatozoa) will not form.
Q5. Which cells in seminiferous tubules provide nutrition to developing sperm?
a) Leydig cells
b) Sertoli cells
c) Spermatogonia
d) Myoid cells
Explanation & Answer:
Correct answer: b) Sertoli cells.
Sertoli cells act as nurse cells, supporting spermatogenesis by supplying nutrients, secreting growth factors, and forming the blood-testis barrier. They also phagocytose residual cytoplasm during spermiogenesis.
Q6. Which phase of meiosis is the longest in spermatogenesis?
a) Prophase I
b) Metaphase I
c) Anaphase II
d) Telophase II
Explanation & Answer:
Correct answer: a) Prophase I.
Prophase I is prolonged, allowing for homologous chromosome pairing, crossing over, and genetic recombination. This increases genetic diversity in gametes and is critical in spermatogenesis.
Q7 (Clinical). A sperm sample reveals only diploid cells. Which stage is likely impaired?
a) Spermatogonial mitosis
b) Meiosis I
c) Spermiogenesis
d) Capacitation
Explanation & Answer:
Correct answer: b) Meiosis I.
Without meiosis I, primary spermatocytes cannot produce haploid secondary spermatocytes, resulting in only diploid cells being present in the semen analysis.
Q8. Which part of the sperm contains the haploid nucleus?
a) Head
b) Neck
c) Midpiece
d) Tail
Explanation & Answer:
Correct answer: a) Head.
The sperm head contains the haploid nucleus, which carries paternal genetic material, and the acrosome, which contains enzymes for penetrating the ovum's zona pellucida during fertilization.
Q9. Which hormone directly stimulates spermatogenesis?
a) FSH
b) LH
c) Testosterone
d) Inhibin
Explanation & Answer:
Correct answer: a) FSH.
FSH acts on Sertoli cells, promoting the process of spermatogenesis. LH indirectly supports spermatogenesis by stimulating Leydig cells to produce testosterone.
Q10. How many functional sperm are produced from one primary spermatocyte?
a) 1
b) 2
c) 3
d) 4
Explanation & Answer:
Correct answer: d) 4.
One primary spermatocyte undergoes meiosis I and II to produce four haploid spermatids, which mature into four spermatozoa during spermiogenesis.
Q11. Sperm chromosome faster is:
a) X chromosome
b) Y chromosome
c) Both same
d) None
Explanation & Answer:
Correct answer: b) Y chromosome.
Y-bearing sperm are lighter and often swim faster due to smaller DNA content compared to X-bearing sperm. However, they may have reduced longevity in the female tract, affecting the timing of conception.
Chapter: Reproductive System
Topic: Male Reproductive Physiology
Subtopic: Sperm Chromosomes
Keyword Definitions:
Sperm: Male gamete responsible for fertilization.
X chromosome: Larger sex chromosome carrying more genes.
Y chromosome: Smaller sex chromosome determining male sex.
Motility: Ability of sperm to swim toward the ovum.
Chromosomal sex determination: Process where X or Y sperm decides offspring’s sex.
Lead Question – 2012
Sperm chromosome faster is -
a) X chromosome
b) Y chromosome
c) Both same
d) None
Explanation: Y chromosome-bearing sperms are lighter and move faster due to their smaller size, whereas X sperms are heavier but live longer. Therefore, Answer: (b) Y chromosome. Clinically, this helps explain sex selection probabilities but natural fertilization remains random. Y sperms reach the ovum quicker but may die sooner.
Question 2. Which chromosome-bearing sperm has a longer lifespan in the female reproductive tract?
a) X chromosome
b) Y chromosome
c) Both same
d) None
Explanation: X chromosome sperms are larger, more resilient, and survive longer (up to 72 hours), while Y sperms die earlier. Thus, Answer: (a) X chromosome. This explains timing-based conception theories clinically, although fertilization chances remain probabilistic.
Question 3. Which type of sperm is more likely to fertilize if intercourse happens 2-3 days before ovulation?
a) X chromosome sperm
b) Y chromosome sperm
c) Both equal
d) None
Explanation: Since X sperms live longer in the cervical mucus, they are more likely to fertilize when intercourse is days before ovulation. Hence, Answer: (a) X chromosome sperm. Clinically linked to natural family planning theories.
Question 4. Which type of sperm is more heat sensitive?
a) X sperm
b) Y sperm
c) Both equal
d) None
Explanation: Y chromosome sperms are more fragile and heat-sensitive, reducing their survival in unfavorable environments. Answer: (b) Y sperm. Clinically important in infertility where scrotal heat affects sperm survival.
Question 5. Which sperm chromosome has higher DNA content?
a) X sperm
b) Y sperm
c) Both equal
d) None
Explanation: X sperms carry larger DNA content as the X chromosome has more genetic material than the smaller Y chromosome. Answer: (a) X sperm. Clinically, this explains slight size differences seen under advanced imaging.
Question 6. Which sperm is more likely to produce a male child?
a) X sperm
b) Y sperm
c) Both equal
d) None
Explanation: Fertilization by Y sperm results in XY (male) offspring, while X sperm results in XX (female). Answer: (b) Y sperm. Clinical relevance in genetic counseling and sex-linked inheritance discussions.
Question 7. Which factor favors fertilization by Y sperms?
a) Acidic cervical mucus
b) Alkaline cervical mucus
c) Neutral pH
d) None
Explanation: Y sperms survive better in alkaline conditions, which favor their motility and fertilizing ability. Answer: (b) Alkaline cervical mucus. Clinical relevance in infertility treatments.
Question 8. Which sperm is slower but survives longer in the female tract?
a) X sperm
b) Y sperm
c) Both equal
d) None
Explanation: X sperms are slower but have greater survival ability, lasting up to 3 days, unlike Y sperms. Answer: (a) X sperm. Useful in understanding natural conception probabilities.
Question 9. Which sperm is more vulnerable to vaginal acidic pH?
a) X sperm
b) Y sperm
c) Both equal
d) None
Explanation: Y sperms are more fragile and less tolerant to acidic environments, making them more vulnerable. Answer: (b) Y sperm. Clinical importance in cases of unexplained infertility due to pH imbalance.
Question 10. In assisted reproduction, which sperm selection may reduce sex-linked disease inheritance in males?
a) X sperm selection
b) Y sperm selection
c) Both equal
d) None
Explanation: Selecting X sperms reduces the chance of sex-linked diseases (e.g., hemophilia, Duchenne muscular dystrophy) since these occur in males (XY). Answer: (a) X sperm selection. Clinical use in IVF with sperm sorting.
Question 11. Which sperm is responsible for sex determination of offspring?
a) X sperm
b) Y sperm
c) Both
d) None
Explanation: Female ovum always carries X chromosome, so the sperm (X or Y) determines the child’s sex. Answer: (c) Both. Clinically used in counseling families with misconceptions about sex determination.
Subtopic: Wolffian Duct Derivatives
Keywords & Definitions:
Wolffian duct: Also known as mesonephric duct; embryonic structure that forms male reproductive tract parts.
Appendix of testis: Remnant of Mullerian duct near testis.
Uterus: Female reproductive organ derived from Mullerian duct.
Appendix of epididymis: Small stalked appendage on epididymis, derived from Wolffian duct.
Hydatid of Morgagni: Cystic remnant, usually appendix testis or epididymis.
Mullerian duct: Embryonic precursor to female reproductive tract.
Embryology: Study of prenatal development.
Sex differentiation: Development of male or female reproductive structures.
Remnant: Vestigial structure from embryonic development.
Mesonephric duct: Synonym for Wolffian duct.
Lead Question - 2012:
Which is derived from Wolffian duct?
a) Appendix of testis
b) Uterus
c) Appendix of epididymis
d) Hydatid of margagni
Explanation & Answer:
The correct answer is c) Appendix of epididymis. The appendix of epididymis is a remnant of the Wolffian duct, which gives rise to male internal genital structures such as the epididymis and vas deferens. The appendix of testis and uterus derive from the Mullerian duct. Hydatid of Morgagni typically refers to either appendix testis or epididymis but is not a direct Wolffian derivative.
Q2. The Mullerian duct develops into which structure?
a) Vas deferens
b) Uterus
c) Epididymis
d) Seminal vesicle
Explanation & Answer:
The Mullerian duct forms the female reproductive tract including uterus, fallopian tubes, and upper vagina. In males, it regresses under anti-Mullerian hormone influence.
Q3. Which hormone induces regression of Mullerian ducts in males?
a) Testosterone
b) Anti-Mullerian hormone
c) Luteinizing hormone
d) Follicle-stimulating hormone
Explanation & Answer:
Anti-Mullerian hormone (AMH), secreted by Sertoli cells, causes regression of Mullerian ducts in male embryos, enabling development of male reproductive tract.
Q4 (Clinical). Persistent Mullerian duct syndrome results in:
a) Presence of uterus in males
b) Absence of vas deferens
c) Absence of epididymis
d) Normal male reproductive tract
Explanation & Answer:
Failure of AMH function causes Persistent Mullerian duct syndrome, where male individuals retain uterus and fallopian tubes, often leading to infertility and cryptorchidism.
Q5. Which structure arises from the Wolffian duct?
a) Seminal vesicle
b) Fallopian tube
c) Uterine tube
d) Clitoris
Explanation & Answer:
The seminal vesicle, part of the male reproductive system, develops from the Wolffian duct. Female structures such as fallopian tubes arise from the Mullerian duct.
Q6. The appendix testis is a remnant of:
a) Wolffian duct
b) Mullerian duct
c) Urogenital sinus
d) Genital tubercle
Explanation & Answer:
Appendix testis is a remnant of the Mullerian duct, located near the testis, differing from appendix epididymis, which is from Wolffian duct.
Q7. The vas deferens develops from:
a) Wolffian duct
b) Mullerian duct
c) Ureteric bud
d) Cloaca
Explanation & Answer:
The vas deferens is derived from the Wolffian duct and serves as the conduit for sperm transport in males.
Q8 (Clinical). Torsion of hydatid of Morgagni presents with:
a) Acute scrotal pain
b) Asymptomatic swelling
c) Urinary retention
d) Fever
Explanation & Answer:
Torsion of the hydatid of Morgagni, a cystic remnant of the testis or epididymis, causes sudden scrotal pain mimicking testicular torsion and requires clinical attention.
Q9. Which is NOT a Wolffian duct derivative?
a) Epididymis
b) Seminal vesicle
c) Uterus
d) Vas deferens
Explanation & Answer:
The uterus is a Mullerian duct derivative; all other options derive from the Wolffian duct and contribute to the male reproductive tract.
Q10. Which hormone promotes Wolffian duct development in males?
a) Estrogen
b) Testosterone
c) Progesterone
d) Anti-Mullerian hormone
Explanation & Answer:
Testosterone stimulates the Wolffian ducts to develop into male internal genitalia such as epididymis, vas deferens, and seminal vesicles during male differentiation.